Buy BOOKS at Discounted Price

Arithmetic Progressions

Class 10th Mathematics RD Sharma Solution
Exercise 9.1
  1. Write the first five terms of each of the following sequences whose nth terms…
  2. Find the indicated terms in each of the following sequences whose nth terms are:…
  3. Find the next five terms of each of the following sequences given by: (i) a_1 =…
Exercise 9.2
  1. For the following arithmetic progressions write the first term a and the common…
  2. Write the arithmetic progression when first term a and common difference of d…
  3. In which of the following situation, the sequence of numbers formed will form an…
  4. Show that the sequence defined by a_n = 5n-7 is an A.P., find its common…
  5. Show that the sequence defined by a_n = 3n^2 - 5 is not A.P.
  6. The general term of a sequence is given by a_n = - 4n^2 + 15 .Is the sequence an…
  7. Find the common difference and write the next four terms of each of the…
  8. Prove that no matter what is the real number a and b are, the sequence with nth…
  9. Write the sequence with nth term: (i) a_n = 3+4n (ii) a_n = 5+2n (iii) a_n = 6-n…
  10. Find out which of the following sequences are arithmetic progressions. For…
  11. Find the common difference of the A.P. and write the next two terms: (i) 51 ,…
  12. The nth term of an A.P. is 6n+2. Find the common difference.
Exercise 9.3
  1. Find: (i) 10th term of the A.P. 1 , 4 , 7 , 10 , (ii) 18th term of the A.P. root…
  2. (i) Which term of the A.P. 3 , 8 , 13 , is 248?
  3. Which term of the A.P. 84 , 80 , 76 is 0?
  4. Which term of the A.P. 4 , 9 , 14 , is 254?
  5. Which term of the A.P. 21 , 42 , 63 , 84 , is 420?
  6. Which term of the A.P. 121 , 117 , 113 , is its first negative term?…
  7. Is -150 a term of the A.P. 11 , 8 , 5 , 2 , ?
  8. Is 68 a term of the A.P. 7 , 10 , 13 , ?
  9. Is 302 a term of the A.P. 3 , 8 , 13 , ?
  10. How many terms are there in the A.P.? (i) 7 , 10 , 13 , 43 (ii) -1 , - 5/6 , -…
  11. The first term of an A.P. is 5, the common difference is 3 and the last term is…
  12. The 6th and 17th terms of an A.P. are 19 and 41 respectively, find the 40th…
  13. If 9th term of an A.P. is zero, prove that its 29th term is double the 19th…
  14. If 10 times the 10th term of an A.P. is equal to 15 times the 15th term, show…
  15. The 10th and 18th terms of an A.P. are 41 and 73 respectively. Find 26th term.…
  16. In a certain A.P. the 24th term is twice the 10th term. Prove that the 72nd…
  17. If (m+1)th term of an A.P. is twice the (n+1)th term, prove that (3m+1)th term…
  18. If the nth term of the A.P. 9 , 7 , 5 , is same as the nth term of the A.P. 15…
  19. Find the 12th term from the end of the following arithmetic progressions: (i) 3…
  20. The 4th term of an A.P. is three times the first and the 7th term exceeds twice…
  21. Find the second term and nth term of an A.P. whose 6th term is 12 and the 8th…
  22. How many numbers of two digit are divisible by 3?
  23. An A.P. consists of 60 terms. If the first and the last terms be 7 and 125…
  24. The sum of 4th and 8th terms of an A.P. is 24 and the sum of the 6th and 10th…
  25. The first term of an A.P. is 5 and its 100th term is - 292. Find the 50th term…
  26. Find a_30-a_20 for the A.P. (i) -9 ,-14 ,-19 ,-24 , (ii) a_1a+d_1s+2d_1s+3d_n…
  27. Write the expression a_n-a_k for the A.P. a_1a+d , a+2d , Hence, find the…
  28. Find n if the given value of x is the nth term of the given A.P. (i) 25 , 50 ,…
  29. If an A.P. consists of n terms with first term a and nth term ls how that the…
  30. Find the arithmetic progression whose third term is 16 and seventh term exceeds…
  31. The 7th term of an A.P. is 32 and its 13th term is 62. Find the A.P.…
  32. Which term of the A.P. 3 , 10 , 17 , will be 84 more than its 13th term?…
  33. Two arithmetic progressions have the same common difference. The difference…
  34. For what value of n, the nth terms of the arithmetic progressions 63 , 65 57 ,…
  35. How many multiples of 4 lie between 10 and 250?
  36. How many three digit numbers are divisible by 7?
  37. Which term of the arithmetic progression 8 , 14 , 20 , 26 , will be72 more than…
  38. Find the term of the arithmetic progression 9 , 12 , 15 , 18 , which is 39 more…
  39. Find the 8th term from the end of the A.P. 7 , 10 , 13 , , 184
  40. Find the 10th term from the end of the A.P. 8 , 10 , 12 , , 126 .…
  41. The sum of 4th and 8th terms of an A.P. is 24 and the sum of 6th and 10th terms…
  42. Which term of the A.P. 3 , 15 , 27 , 39 , will be 120 more than its 21st term?…
  43. The 17th term of an A.P. is 5 more than twice its 8th term. If the 11th term of…
  44. Find the number of all three digit natural numbers which are divisible by 9.…
  45. The 19th term of an A.P. is equal to three times its sixth term. If its 9th…
  46. The 9th term of an A.P. is equal to 6 times its second term. If its 5th term is…
  47. The 24th term of an A.P. is twice its 10th term. Show that its 72nd term is 4…
  48. Find the number of natural numbers between 101 and 999 which are divisible by…
  49. If the seventh term of an A.P. is 1/9 and its ninth term is1/7, find its (63)rd…
  50. The sum of 5th and 9th terms of an AP is 30. If its 25th term is three times…
  51. Find where 0 (zero) is a term of the A.P. 40 , 37 , 34 , 31 ,
  52. Find the middle term of the A.P. 213 , 205 , 197 , , 37 .
  53. If the 5th term of an A.P. is 31 and 25th term is 140 more than the 5th term,…
Exercise 9.4
  1. The sum of three terms of an A.P. is 21 and the product of the first and the…
  2. Three numbers are in A.P. If the sum of these numbers be 27 and the product 648,…
  3. Find the four numbers in A.P. whose sum is 50 and in which the greatest number…
  4. The angles of a quadrilateral are in A.P. whose common difference is 10. Find…
  5. The sum of three numbers in A.P. is 12 and the sum of their cube is 288. Find…
  6. Find the value of x for which (8x+4), (6x-2), (2x+7) are in A.P.
  7. If x+1, 3x and 4x+2 are in A.P, find the value of x.
  8. Show that (a-b)^2 , (a^2 +b^2) and (a + b)^2 are in A.P.
Exercise 9.5
  1. Find the sum of the following arithmetic progressions: (i) 50 , 46 , 42 , to 10…
  2. Find the sum on n term of the A.P. 5 , 2 ,-1 ,-4 ,-7 , ,
  3. Find the sum of n terms of an A.P. whose nth term is given by a_n = 5-6n…
  4. If the n sum of a certain number of terms starting from first term of an A.P. is…
  5. How many terms of the sequence 18 , 16 , 14 , should be taken so that their sum…
  6. How many terms are there in the A.P. whose first and fifth terms are 14 and 2…
  7. How many terms of the A.P. 9 , 17 , 25 , must be taken so that their sum is…
  8. How many terms of the A.P. 63 , 60 , 57 , must be taken so that their sum is…
  9. The first and the last terms of an A.P. are 17 and 350 respectively. If the…
  10. The third term of an A.P. is 7 and the seventh term exceeds three times the…
  11. The first term of an A.P. is 2 and the last term is 50. The sum of all these…
  12. If 12th term of an A.P. is 13 and the sum of the first four terms is 24, what is…
  13. Find the sum of first 22 terms of an A.P. in which d = 22 and a22 = 149.…
  14. Find the sum of all natural numbers between a and 100, which are divisible by…
  15. Find the sum of first n odd natural numbers.
  16. Find the sum of all odd numbers between (i) 0 and 50 (ii) 100 and 200…
  17. Show that the sum of all odd integers between 1 and 1000 which are divisible by…
  18. Find the sum of all integers between 84 and 719, which are multiples of 5.…
  19. Find the sum of all integers between 50 and 500, which are multiples of 7.…
  20. Find the sum of all even integers between 101 and 999.
  21. Find the sum of all integers between 100 and 550, which are multiples of 9.…
  22. In an A.P. if the first term is 22, the common difference is - 4 and the sum to…
  23. In an A.P. If the 5th and 12th terms are 30 and 65 respectively, what is the…
  24. Find the sum of the first (i) 11 terms of the A.P : 2 , 6 , 10 , 14 , (ii) 13…
  25. Find the sum of (i) The first 15 multiples of 8 (ii) The first 40 positive…
  26. Find the sum : (i) 2+4+6 + l +200 (ii) 3+11+19 + l +803 (iii) (-5) + (-8) +…
  27. Find the sum of the first 15 terms of each of the following sequences having…
  28. Find the sum of first 20 terms of these sequence whose nth term is a_n = an+b .…
  29. Find the sum of first 25 terms of an A.P. whose nth term is given by a_n = 2-3n…
  30. Find the sum of the first 25 terms of an A.P. whose nth term is given by a_n =…
  31. Find the sum of the first 25 terms of an A.P. whose second and third terms are…
  32. If the sum of 7 terms of an A.P. is 49 and that of 17 term is 289, find the sum…
  33. The first term of an A.P. is 5, the last term is 45 and the sum is 400. Find…
  34. In an A.P., the sum of first n terms is 3n^2/2 + 13/0 n . Find its 25th term.…
  35. Let there be an A.P. with first term a, common difference, d. If a_n denotes…
  36. Aman saved Rs. 16500 in ten years. In each year after the first he saved Rs.100…
  37. Aman saved Rs.32 during the first year, Rs.36 in the second year and in this…
  38. Aman arrange stop ay off a debt of Rs.3600 by 40 annual installments which for…
  39. There are 25 trees at equal distances of 5 metres in a line with a well, the…
  40. A man is employed to count Rs.10710. He counts at there of Rs.180 per minute…
  41. A piece of equipment cost a certain factory Rs.600,000. If it depreciates in…
  42. A sum of Rs.700 is to be used to gives even cash prizes to students of a school…
  43. In an A.P. the first term is 8, nth term is 33 and the sum to first n terms is…
  44. In an A.P., the first term is 22, nth term is -11 and the sum to first n terms…
  45. If the sum of the first n terms of an A.P. is 4n - n^2 , which is the first…
  46. The first and the last term of an A.P. are 17 and 350 respectively. If the…
  47. In an A.P., the first term is 2, the last term is 29 and the sum of the terms…
  48. In an A.P., the sum of first ten terms is 150 and the sum of its next ten terms…
  49. Sum of the first 14 terms of an A.P. is 1505 and its first term is 10. Find its…
  50. The sum of first n terms of an A.P. is 5n^2 + 3n . If its mth term is 168, find…
  51. The sum of first q terms of an A.P. is 63q - 3q^2 . If its pth term is-60, find…
  52. The sum of first m terms of an A.P. is 4 m^2 - m. If its nth term is 107, find…
  53. The nth term of an A.P. is given by (-4n+15). Find the sum of first 20 terms of…
  54. Find the number of terms of the A.P. If 1 is added to each term of this A.P.,…
  55. The sum of first n terms of an A.P. is 3n^2 + 4n. Find the 25th term of this…
  56. In a school, students decide to plant trees in and around the school to reduce…
  57. The sum of first seven terms of an A.P. is 182. If its 4th and the 17th terms…
  58. The sum of the first n terms of an A.P. is 3n^2 + 6n. Find the nth term of this…
  59. The sum of the first 7 terms of an A.P. is 63 and the sum of its next 7 terms…
  60. If demotes the sum of the first n terms of an A.P., prove that S30= 3 (S20 -…
  61. The sum of first n terms of an A.P. is 5n - n^2 . Find the nth term of this…
  62. The sum of the first n terms of an A.P. is 4n^2 + 2n. Find the nth term of this…
  63. If the 10th term of an A.P. is 21 and the sum of its first ten terms is 120,…
  64. Ram kali would need Rs.1800 for admission fee and books etc., for her daughter…
  65. The first and the last terms of an A.P. are 5 and 45 respectively. If the sum…
  66. The first and the last terms of an AP are 7 and 49 respectively. If sum of all…
  67. If demotes the sum of first n terms of an A.P., prove that S12 = 3 (s8 - S4) .…
  68. If the sum of first n terms of an A.P. is 1/2 (3n^2 + 7n), then find its nth…
  69. The sum of first 9 terms of an A.P. is 162. The ratio of its 6th term to its…
Cce - Formative Assessment
  1. Define an arithmetic progression.
  2. If 7th and 13th terms of an A.P. be 34 and 64 respectively, then its 18th term isA. 87…
  3. Write the common difference of an A.P. whose nth term is an = 3n + 7.…
  4. If the sum of P terms of an A.P. is q and the sum of q terms is p, then the sum of p +…
  5. Which tem of the sequence 114, 109, 104, .... is the first negative term?…
  6. If the sum of n terms of an A.P. be 3n^2 + n and its common difference is 6, then its…
  7. The first and last terms of an A.P. are 1 and 11. If the sum of its terms is 36, then…
  8. Write the value of a30 - a10 for the A.P. 4, 9, 14, 19, ……….
  9. If the sum of n terms of an A.P. is 3n^2 + 5n then which of its terms is 164?A. 26th B.…
  10. Write 5th term from the end of the A.P. 3, 5, 7, 9, ...., 201.
  11. If the sum of n terms of an A.P. is 2n^2 + 5n, then its nth term isA. 4n - 3 B. 3n - 4…
  12. Write the value of x for which 2x, x + 10 and 3x + 2 are in A.P.
  13. If the sum of three consecutive terms of an increasing A.P. is 51 and the product of…
  14. Write the nth term of an A.P. the sum of whose n terms is Sn.
  15. If four numbers in A.P. are such that their sum is 50 and the greatest number is 4…
  16. Write the sum of first n odd natural numbers.
  17. Let Sn denote the sum of n terms of an A.P. whose first term is a. If the common…
  18. Write the sum of first n even natural numbers.
  19. The first and last term of an A.P. are a and l respectively. If S is the sum of all…
  20. 1f the sum of n terms of an A.P. is Sn = 3n^2 + 5n. Write its common difference.…
  21. Write the expression for the common difference of an A.P. whose first term is a and…
  22. If the sum of first n even natural numbers is equal to k times the sum of first n odd…
  23. If the first, second and last term of an A.P. are a, b and 2a respectively, its sum…
  24. The first term of an A.P. is p and its common difference is q. Find its 10th term.…
  25. For what values of p are 2p + 1, 13, 5p - 3 are three consecutive terms of an A.P.?…
  26. If S1 is the sum of an arithmetic progression of 'n' odd number of terms and S2 the…
  27. If 4/5 , a, 2 are three consecutive terms of an A.P., then find the value of a.…
  28. If in an A.P., Sn = n^2 p and Sm = m^2 p, where Sr denotes the sum of r terms of the…
  29. If Sn denote the sum of the first n terms of an A.P. If S2n = 3Sn, then S3n : Sn is…
  30. If the sum of first p term of an A.P. is ap^2 + bp, find its common difference.…
  31. In an AP, Sp = q, Sq = p and Sr denotes the sum of first r terms. Then, Sp + q is…
  32. If Sr denotes the sum of the first r terms of an A.P. Then, S3n: (S2n — Sn) isA. n B.…
  33. If the first term of an A.P. is 2 and common difference is 4, then the sum of its 40…
  34. The number of terms of the A.P. 3, 7, 11, 15, ... to be taken so that the sum is 406…
  35. Sum of n terms of the series root 2 + root 8 + root 18 + root 32+.. isA. n (n+1)/2 B.…
  36. The 9th term of an A.P. is 449 and 449th term is 9. The term which is equal to zero…
  37. If 1/x+2 , 1/x+3 , 1/x+5 are in A.P. Then, x =A. 5 B. 3 C. 1 D. 2…
  38. The nth term of an A.P., the sum of whose n terms is Sn, isA. Sn + Sn-1 B. Sn - Sn-1…
  39. The common difference of an A.P., the sum of whose n terms is Sn, isA. Sn - 2Sn-1 +…
  40. If the sums of n terms of two arithmetic progressions are in the ratio 3n+5/5n+7 ,…
  41. If Sn, denote the sum of n terms of an A.P. with first term a and common difference d…
  42. If the first term of an A.P. is a and nth term is b, then its common difference isA.…
  43. The sum of first n odd natural numbers isA. 2n - 1 B. 2n + 1 C. n^2 D. n^2 - 1…
  44. Two A.P.'s have the same common difference. The first term of one of these is 8 and…
  45. If 18, a, b, -3 are in A.P., the a + b =A. 19 B. 7 C. 11 D. 15
  46. The sum of n terms of two A.P.'s are in the ratio 5n + 9 : 9n + 6. Then, the ratio of…
  47. If , then n =A. 8 B. 7 C. 10 D. 11
  48. The sum of n terms of an A.P. is 3n^2 + 5n, then 164 is itsA. 24th term B. 27th term…
  49. If the nth term of an A.P. is 2n + 1, then the sum of first n terms of the A.P. isA. n…
  50. If 18th and 11th term of an A.P. are in the ratio 3 : 2, then its 21st and 5th terms…
  51. The sum of first 20 odd natural numbers isA. 100 B. 210 C. 400 D. 420…
  52. The common difference of the A.P. is 1/2q , 1-2q/2q , 1-4q/2q , l , isA. -1 B. 1 C. q…
  53. The common difference of the A.P. 1/3 , 1-3b/3 , 1-6b/3 , isA. 1/3 B. - 1/3 C. -b D. b…
  54. The common difference of the A.P. 1/2b , 1-6b/2b , 1-12b/2b , isA. 2b B. -2b C. 3 D.…
  55. If k, 2k -1 and 2k + 1 are three consecutive terms of an AP, the value of k isA. - 2…
  56. The next term of the A.P. root 7 , root 28 , root 63 , A. root 70 B. root 84 C. root…
  57. The first three terms of an A.P. respectively are 3y - 1, 3y + 5 and 5y + 1. Then, y…

Exercise 9.1
Question 1.

Write the first five terms of each of the following sequences whose nth terms are :

(i)

(ii)

(iii)

(iv)

(v)

(vi)

(vii)

(viii)

(ix)


Answer:

(i)


Put n = 1


A1 = 3(1) + 2 = 3+2=5


Put n = 2


A2 = 3(2) + 2=8


Put n = 3


A3 = 3(3) + 2 = 9+2=11


Put n = 4


A4 = 3(4) +2 = 12+2=14


Put n =5


A5 = 3(5) + 2 = 15+2=17


(ii)


Put n = 1


A1= =


Put n = 2


A2= = 0


Put n = 3


A3= =


Put n = 4


A4= =


Put n = 5


A5= = = 1


(iii)


Put n = 1


A1= 31= 3


Put n = 2


A2= 32= 9


Put n = 3


A3= 33= 27


Put n = 4


A4= 34= 81


Put n = 5


A5= 35= 243


(iv)


Put n = 1


A1= = =


Put n = 2


A2= = =


Put n = 3


A3= = =


Put n = 4


A4= = = = 2


Put n = 5


A5 = = =


(v)


Put n = 1


A1= (-1)1.21 = (-1).2 = -2


Put n = 2


A2= (-1)2.22 = (1).4 = 4


Put n = 3


A3= (-1)3.23 = (-1).8 = -8


Put n = 4


A4= (-1)4.24 = (1).16 = 16


Put n = 5


A5= (-1)5.25 = (-1).32 = -32


(vi)


Put n = 1


A1= =


Put n = 2


A2= = 0


Put n = 3


A3= =


Put n = 4


A4= = = 4


Put n = 5


A5= =


(vii)


Put n = 1


A1= (1)2 – 1+1 = 1


Put n = 2


A2= (2)2 -2+1 = 3


Put n = 3


A3= (3)2 -3+1 = 9-2 = 7


Put n = 4


A4 = (4)2 -4+1 = 16-3 = 13


Put n = 5


A5= (5)2 – 5+1 = 25-4 = 21


(viii)


Put n = 1


A1= 2(1)2– 3(1) + 1 = 2-3+1 = 0


Put n = 2


A2= 2(2)2 – 3(2) + 1 = 8-6+1 = 3


Put n = 3


A3= 2(3)2 – 3(3) + 1 = 18-9+1 = 10


Put n = 4


A4= 2(4)2 – 3(4) + 1 = 32-12+1 = 21


Put n = 5


A5= 2(5)2 – 3(5) + 1 = 50-15+1 = 36


(ix)


Put n = 1


A1= =


Put n = 2


A2= =


Put n = 3


A3= = =


Put n = 4


A4= =


Put n = 5


A5= =


Question 2.

Find the indicated terms in each of the following sequences whose nth terms are:

(i)

(ii)

(iii)

(iv)

(v)


Answer:

(i)


Put n = 12


A12= 5(12) – 4


= 60-4 = 56


Put n = 15


A15= 5(15) – 4


= 75-4 = 71


(ii)


Put n = 7


A7= = =


Put n = 8


A8= = =


(iii)


Put n = 5


A5= 5(5-1) (5-2)


= 5(4)(3) = 60


Put n = 8


A8= 8(8-1) (8-2)


= 8(7)(6) = 336


(iv)


Put n = 1


A1= 1(1-1) (2-1) (2-2) (3+2)


= 0


Put n = 2


A2= 2(2-1) (2-2) (3+2)


= 0


Put n = 3


A3= 3(3-1) (2-3) (3+3)


= 3 (2) (-1) (6)


= 36


(v)


Put n = 3


A3= (-1)3 (3) = -3


Put n = 5


A5= (-1)5 (5) = -5


Put n = 8


A8= (-1)8 (8) = 8



Question 3.

Find the next five terms of each of the following sequences given by:

(i)

(ii)

(iii)

(iv)


Answer:

(i)


Put n = 2


A2= a2-1 + 2


A2= a1 +2


= 1+2 = 3


Put n = 3


A3= a3-1 +2


= a2 + 2


= 3+2 = 5


Put n = 4


A4= a4-1 + 2


= a3 + 2


= 5+2 = 7


Put n = 5


A5= a5-1 + 2


= a4 + 2


= 7+2 = 9


Put n = 6


A6 = a6-1 +2


= a5 +2


= 9+2 = 11


(ii)


Put n = 3


A3= a3-1 – 3


= a2 – 3


= 2-3 = -1


Put n = 4


A4= a4-1 – 3


= a3 – 3


= -1 – 3 = -4


Put n = 5


A5 = a5-1 -3


= a4 – 3


= -4 – 3 = -7


Put n = 6


A6= a6-1 – 3


= a5 – 3


= -7 – 3 = -10


Put n = 7


A7= a7-1 – 3


= a6 – 3


= -10 – 3 = -13


(iii)


Put n = 2


A2= a2-1 / 2


= -1/2


Put n = 3


A3= a3-1 / 3


= -1/6


Put n = 4


A4 = a4-1 / 3


= -1/24


Put n = 5


A5= a5-1 / 3


= -1/120


Put n = 6


A6= a6-1 / 3


= -1/720


(iv)


Put n = 2


A2= 4a2-1 + 3


= 4a1 + 3 = 19


Put n = 3


A3= 4a3-1 +3


= 4a2 + 3 = 79


Put n = 4


A4 = 4a4-1 + 3


= 4a3 + 3


= 316 + 3


= 319


Put n = 5


A5 = 4a5-1 + 3


= 4a4 + 3


= 1276 + 3


= 1279


Put n = 6


A6= 4a6-1 + 3


= 4a5 + 3


= 5116 + 3


= 5119




Exercise 9.2
Question 1.

For the following arithmetic progressions write the first term a and the common difference d :

(i)

(ii)

(iii)

(iv)


Answer:

(i)


First term, a = -5


Common difference, d = -1 – (-5)


= -1 + 5 = 4


(ii)


First term, a =


Common difference, d = - =


(iii)


First term, a = 0.3


Common difference, d = 0.55 – 0.30 = 0.25


(iv)


First term, a = -1.1


Common difference, d = -3.1 – (-1.1)


= -2



Question 2.

Write the arithmetic progression when first term a and common difference of d are as follows:

(i)

(ii)

(iii)


Answer:

(i)


a1= 4, d= -3


A2= a1+ d = 4-3 =1


A3= a1+ 2d = 4+ 2(-3) = 4-6 = -2


A4= a1 =3d = 4+ 3(-3) = 4-9 = -5


Series = 4, 1, -2, -5,…


(ii)


a1= -1, d =


A2= a1 + d = -1 + =


A3= a1 + 2d = -1 + 1 = 0


A4= a1 + 3d = -1 + =


Series = -1, , 0, ,…


(iii)


a1= -1.5, d= -0.5


A2= a1+ d = -1.5 – 0.5 = -2


A3= a1+ 2d = -1.5 + 2(-0.5) = -1.5 – 1 = -2.5


A4= a1+ 3d = -1.5 + 3(-0.5) = -1.5 – 1.5 = -3


Series =



Question 3.

In which of the following situation, the sequence of numbers formed will form an A.P.?

(i) The cost of digging a well for the first metre is Rs. 150 and rises by Rs.20 for each succeeding metre.

(ii)The mount of air present in the cylinder when a vacuum pump removes each time of their remaining in the cylinder.


Answer:

(i) The cost of digging a well for the first metre = Rs. 150


The cost of digging a well for the second metre = 150 + 20 = Rs. 170


The cost of digging a well for the third metre = 170 + 20 = Rs. 190


The cost of digging a well for the fourth metre = 190 + 20 = Rs. 210


Difference between first and second metre = 170 – 150 = 20


Difference between second and third metre = 190 – 170 = 20


Difference between third and fourth metre = 210 – 190 = 20


Since, all the differences are equal. Therefore, it’s an A.P.


(ii) Let the amount of air present in the cylinder first time = x


Amount of air present in the cylinder second time = x - =


Amount of air present in the cylinder third time = - =


Difference in the amount of air present in the cylinder between first and the second time = – x =


Difference in the amount of air present in the cylinder between second and the third time = -


=


Since, the differences are unequal. Therefore, it’s not an A.P.



Question 4.

Show that the sequence defined byis an A.P., find its common difference.


Answer:

Put n = 1

A1= 5(1) – 7


= -2


Put n = 2


A2= 5(2) – 7


= 10 – 7 = 3


Put n = 3


A3= 5(3) – 7


= 15 – 7 = 8


Common difference, d1= a2 – a1= 3 – (-2) = 5


Common difference, d2= a3 – a2= 8 – 3 = 5


Since, d1 = d2 and Common difference = 5


Therefore, it’s an A.P.



Question 5.

Show that the sequence defined by is not A.P.


Answer:

Given: The sequence .

To prove: the sequence defined by is not A.P.

Proof:

Consider the sequence ,


Put n = 1

A1= 3(1)2 – 5 = 3 – 5 = -2


Put n = 2


A2= 3(2)2 – 5 = 12- 5 = 7


Put n = 3


A3= 3(3)2 – 5 = 27 – 5 = 22

In an A.P the difference of consecutive terms should be same.


So,

Common difference, d1= a2 – a1 =7 – (-2) = 9


Common difference, d2 = a3 – a2 = 22 – 9 = 13


Since, d1 ≠ d2


Therefore, it’s not an A.P.


Question 6.

The general term of a sequence is given by .Is the sequence an A.P..? If so, find its 15th term and the common difference.


Answer:

Put n = 1

A1= -4(1)2 + 15 = 11


Put n = 2


A2= -4(2)2 + 15 = -1


Put n = 3


A3= -4(3)2 + 15 = -21


Common difference, d1= a2 – a1 = -1 – 11 = -12


Common difference, d2= a3 – a2 = -21 + 1 = 20


Since, d1 ≠ d2


Therefore, it’s not an A.P.



Question 7.

Find the common difference and write the next four terms of each of the following arithmetic progressions:

(i)

(ii)

(iii)

(iv)


Answer:

(i)


Common difference, d = -2 – 1 = -3


A5= -8 – 3 = -11


A6= -11 – 3 = -14


A7= -14 – 3 = -17


A8= -17 – 3 = -20


(ii)


Common difference, d = -3 – 0 = -3


A5= -9 – 3 = -12


A6= -12 – 3 = -15


A7= -15 – 3 = -18


A8= -18 – 3 = -21


(iii)


Common difference, d = + 1 =


A4= + =


A5= + = = 4


A6= 4 + =


A7= + = =


(iv)


Common difference, d = + 1 =


A4= + =


A5= + =


A6= + =


A7= + = 0



Question 8.

Prove that no matter what is the real number a and b are, the sequence with nth term a+nb is always an A.P. What is the common difference?


Answer:

Put n = 1

A1= a + b


Put n = 2


A2= a + 2b


Put n = 3


A3= a + 3b


Put n = 4


A4= a + 4b


Common difference, d1 = a2 – a1 = a + 2b – a – b = b


Common difference, d2= a3 – a2 = a+ 3b – a – 2b = b


Common difference, d3 = a4 – a3 = a+ 4b – a – 3b = b


Since, d1 = d2 = d3


Therefore, it’s an A.P. with common difference ‘b’



Question 9.

Write the sequence with nth term:

(i)

(ii)

(iii)

(iv)

Show that all of the above sequences form A.P.


Answer:

(i) Put n = 1

A1= 3 + 4(1) = 7


Put n = 2


A2= 3 + 4(2) = 11


Put n = 3


A3= 3 + 4(3) = 15


Common difference, d1= a2 – a1 = 11 – 7 = 4


Common difference, d2= a3 – a2 = 15 – 11 = 4


Since, d1 = d2


Therefore, it’s an A.P. with sequence 7, 11, 15,…


(ii) Put n = 1


A1= 5 + 2(1) = 7


Put n = 2


A2= 5 + 2(2) = 9


Put n = 3


A3= 5 + 2(3) = 11


Common difference, d1= a2 – a1= 9 – 7 = 2


Common difference, d2= a3 – a2= 11 – 9 = 2


Since, d1 = d2


Therefore, it’s an A.P. with sequence 7, 9, 11,…


(iii) Put n = 1


A1= 6 – 1 = 5


Put n = 2


A2= 6 – 2 = 4


Put n = 3


A3= 6 – 3 = 3


Common difference, d1= a2 – a1= 4 – 5 = -1


Common difference, d2 = a3- a2= 3 – 4 = -1


Since, d1=d2


Therefore, it’s an A.P. with sequence 5, 4 , 3,…


(iv) Put n = 1


A1= 9 – 5(1) = 4


Put n = 2


A2= 9 – 5(2) = -1


Put n = 3


A3= 9 – 5(3) = -6


Common difference, d1= a2 – a1 = -1 – 4 = -5


Common difference,d2= a3 – a2 = -6 – (-1) = -5


Since, d1=d2


Therefore, it’s an A.P. with sequence 4, -1, -6,…



Question 10.

Find out which of the following sequences are arithmetic progressions. For those which are arithmetic progressions, find out the common difference.

(i)

(ii)

(iii)

(iv)

(v)

(vi)

(vii)

(viii)

(ix)

(x)

(xi)

(xii)


Answer:

(i)


Common difference, d1 = 6 – 3 = 3


Common difference, d2= 12 – 6 = 6


Since, d1 ≠ d2


Therefore, it’s not an A.P.


(ii)


Common difference, d1 = -4 – 0 = -4


Common difference, d2= -8 – (-4) = - 4


Since, d1 = d2


Therefore, it’s an A.P. with common difference, d = -4


(iii)


Common difference, d1= - =


Common difference, d2 = - =


Since, d1 ≠ d2


Therefore, it’s not an A.P.


(iv)


Common difference, d1= 2 – 12 = -10


Common difference, d2= -8 -2 = -10


Since,d1 = d2


Therefore, it’s an A.P. with common difference, d = -10


(v)


Common difference, d1= 3 – 3 = 0


Common difference, d2= 3 – 3 = 0


Since, d1=d2


Therefore, it’s an A.P. with common difference, d = 0


(vi)


Common difference, d1= p + 90 – p = 90


Common difference, d2= p + 180 – p – 90 = 90


Since, d1=d2


Therefore, it’s an A.P. with common difference, d = 90


(vii)


Common difference, d1= 1.7 – 1.0 = 0.7


Common difference, d2= 2.4 – 1.7 = 0.7


Since, d1=d2


Therefore, it’s an A.P. with common difference, d = 0.7


(viii)


Common difference, d1= -425 + 225 = -200


Common difference, d2= -625 + 425 = -200


Since, d1=d2


Therefore, it’s an A.P. with common difference, d = -200


(ix)


Common difference, d1= 10 + 26 – 10 = 26 = 64


Common difference, d2 = 10 + 27 – 10 – 26 = 26 (2 – 1) = 64


Since, d1=d2


Therefore, it’s an A.P. with common difference, d = 64


(x)


Common difference, d1 = (a + 1) + b – a – b = 1


Common difference, d2 = (a + 1) + (b + 1) – (a + 1) – b = 1


Since, d1 = d2


Therefore, it’s an A.P. with common difference, d = 1


(xi)


Common difference, d1= 32 – 12 = 8


Common difference, d2 = 52 – 32 = 25 – 9 = 16


Since, d1≠d2


Therefore, it’s not an A.P.


(xii)


Common difference, d1 = 52 – 12 = 24


Common difference, d2 = 72 – 52 = 24


Since, d1 = d2


Therefore, it’s an A.P. with common difference, d = 24



Question 11.

Find the common difference of the A.P. and write the next two terms:

(i)

(ii)

(iii)

(iv)

(v)


Answer:

(i)


Common difference, d = 59 – 51 = 8


A5 = 75 + 8 = 83


A6 = 83 + 8 = 91


(ii)


Common difference, d = 67 – 75 = -8


A5= 51 - 8 = 43


A6= 43 – 8 = 35


(iii)


Common difference, d = 2.0 – 1.8 = 0.2


A5= 2.4 + 0.2 = 2.6


A6= 2.6 + 0.2 = 2.8


(iv)


Common difference, d = – 0 =


A5= + = 4


A6= 4 + =


(v)


Common difference, d = 136 – 119 = 17


A5= 170 + 17 = 187


A6= 187 + 17 = 204



Question 12.

The nth term of an A.P. is 6n+2. Find the common difference.


Answer:

an= 6n + 2

Put n = 1


A1= 6(1) + 2 = 8


Put n = 2


A2= 6(2) + 2 = 14


Common difference, d = a2 – a1 = 14 – 8 = 6




Exercise 9.3
Question 1.

Find:

(i) 10th term of the A.P.

(ii) 18th term of the A.P.

(iii) nth term of the A.P.

(iv) 10th term of the A.P.

(v) 8th term of the A.P.

(vi) 11th term of the A.P.

(vii) 9th term of the A.P.


Answer:

(i) 10th term of the A.P.


a = 1, d = 4 – 1 = 3


A10= a + (10 – 1) d


= 1 + (9)3 = 28


(ii) 18th term of the A.P.


a = √2, d = 3√2 - √2 = 2√2


A18= a + (18 – 1) d


= √2 + (17) 2√2 = 35√2


(iii) nth term of the A.P.


a = 13, d = -5


An= a + (n-1)d = 13 + (n-1) (-5)


= 13 -5n + 5 = 18 – 5n


(iv) 10th term of the A.P.


a = -40, d = 25


A10= a + (10-1) d = -40 + 9 (25)


= -40 + 225 = 185


(v) 8th term of the A.P.


a = 117, d = -13


A8= a + (8 – 1) d = 117 + 7 (-13)


= 117 – 91 = 26


(vi) 11th term of the A.P.


a = 10.0, d = 10.5 – 10.0 = 0.5


A11= a + (11 – 1) d = 10.0 + 10 (0.5)


= 10.0 + 5 = 15.0


(vii) 9th term of the A.P.


a = , d = - =


A9= a + (9 – 1) d = + 8 () =



Question 2.

(i) Which term of the A.P. is 248?


Answer:

a = 3, d = 5

An= a + (n – 1) d


248 = 3 + (n – 1) 5 = 3 + 5n – 5


248 = -2 + 5n


250 = 5n


n = 50



Question 3.

Which term of the A.P.is 0?


Answer:

a = 84, d = -4,An = 0

We know,

An= a + (n – 1) d


0 = 84 + (n – 1) -4


0 = 84 – 4n + 4


0 = 88 – 4n


n = 22


Question 4.

Which term of the A.P. is 254?


Answer:

a = 4, d = 5

An= a + (n – 1) d


254 = 4 + (n – 1) 5


254 = 4 + 5n – 5


254 = -1 + 5n


255 = 5n


n = 51



Question 5.

Which term of the A.P. is 420?


Answer:

a = 21, d = 21

An= a + (n – 1) d


420 = 21 + (n – 1) 21


420 = 21 + 21n – 21


420 = 21n


n = 20



Question 6.

Which term of the A.P. is its first negative term?


Answer:

a = 121, d = -4

An = -ve


An= a + (n – 1) d


= 121 + (n – 1) -4


= 125 – 4n


Since, we want –ve term


Therefore, 4n should be a number greater than 125


Hence, 128 is the number greater than 125 and divisible by 4


4n = 128


n = 32


Therefore, 32nd term of the given A.P. is –ve



Question 7.

Is -150 a term of the A.P. ?


Answer:

a = 11, d = -3

An= a + (n – 1) d


-150 = 11 + (n – 1) (-3)


-150 = 11 – 3n + 3


-150 = 14 – 3n


-164 = 3n


Since, -164 is not divisible by 3


Therefore, -150 isn’t the term of the given A.P.



Question 8.

Is 68 a term of the A.P. ?


Answer:

a = 7, d = 3

An= a + (n – 1) d


68= 7 + (n – 1) 3


68= 4 + 3n


3n = 64


Since, 64 is not divisible by 3


Therefore, 68 isn’t the term of the given A.P.



Question 9.

Is 302 a term of the A.P. ?


Answer:

a = 3, d = 5

An= a + (n – 1) d


302 = 3 + (n – 1) 5


302 = -2 + 5n


304 = 5n


Since, 304 is not divisible by 5


Therefore, 302 isn’t the term of the given A.P.



Question 10.

How many terms are there in the A.P.?

(i)

(ii)

(iii)

(iv)


Answer:

(i)


a = 7, d = 3


An = 43


a + (n – 1) d = 43


7 + (n – 1) 3 = 43


3n = 39


n = 13


Therefore, there are total 13 terms in the A.P.


(ii)


a = -1, d =


An =


a + (n – 1) d =


-1 + (n -1) =


n = 26


(iii)


a = 7, d = 6


An = 205


a + (n – 1) d = 205


7 + (n – 1) 6 = 205


6n = 204


n = 34


(iv)


a = 18, d =


An = -47


a + (n – 1) d = -47


18 + (n – 1) = -47


18 - + = -47


n = 27



Question 11.

The first term of an A.P. is 5, the common difference is 3 and the last term is 80; find the number of terms.


Answer:

Given: The first term of an A.P. is 5, the common difference is 3 and the last term is 80

To find: the number of terms.


Solution :


We have a = 5, d = 3

an = 80

From the formula an = a + (n-1) d


Number of terms is :



= + 1


= + 1


= 25 + 1


= 26

Hence the number of terms in a given A.P. is 26.


Question 12.

The 6th and 17th terms of an A.P. are 19 and 41 respectively, find the 40th term.


Answer:

a6= 19 = a + (n – 1) d

=19 = a + 5d …(i)


A17 = 41 = a + (n – 1) d


= 41 = a + 16d …(ii)


Subtracting (i) from (ii), we get


22 = 11d


d = 2


Now substituting the value of d in (i): 19 = a + 10


= a = 9


A40= a + (40 – 1) 2


= 9 + 78


= 87



Question 13.

If 9th term of an A.P. is zero, prove that its 29th term is double the 19th term.


Answer:

a9 = 0

a + (9 – 1)d =0


a + 8d = 0


a = -8d …(i)


To Prove: a29 = 2a19


Proof: LHS= a29 = a + 28d = -8d + 28d = 20d


RHS= 2a19 = 2[a + (18)d] = 2(-8d + 18d) = 2(10d) = 20d


Since, LHS=RHS


Hence, proved



Question 14.

If 10 times the 10th term of an A.P. is equal to 15 times the 15th term, show that 25th term of the A.P. is zero.


Answer:

10a10 = 15a15

10[a + (10 – 1)d] = 15[a + (15 – 1)d]


2a + 2(9)d = 3a + 3(14)d


-a = 42d – 18d


-a = 24d


a = -24d


a25 = a + 24d


= -24d + 24d


= 0



Question 15.

The 10th and 18th terms of an A.P. are 41 and 73 respectively. Find 26th term.


Answer:

a10 = 41

a + (10 – 1)d = 41


a + 9d = 41 …(i)


a18 = 73


a + (18 – 1)d = 73


Substituting the value of a from (i),


41 – 9d + 17d = 73


8d = 32


d =4


a = 41 – 9(4)


a =41 – 36


a =5


a26 =a + (26 – 1)d


=5 + 25(4)


=5 + 100


=105



Question 16.

In a certain A.P. the 24th term is twice the 10th term. Prove that the 72nd term is twice the 34th term.


Answer:

Given: a24 = 2a10

To Prove: a72 = 2a34


Proof: a24 = 2a10


a + (24 –1)d = 2a + 2(10 –1)d


23d – 18d = a


a = 5d


LHS= a72 =a + 71d =5d +71d =76d


RHS= a34 =2a + 2(33)d =10d + 66d =76d


Since, LHS=RHS


Hence, proved



Question 17.

If (m+1)th term of an A.P. is twice the (n+1)th term, prove that (3m+1)th term is twice (m+n+1)th term.


Answer:

Given:
(m+1)th term of an A.P. is twice the (n+1)th term
a(m +1) =2a(n +1)


To Prove:
(3m+1)th term is twice (m+n+1)th term
a(3m + 1) =2a(m +n +1)

Proof:
a(m +1) =2a(n +1)

⇒ a + (m + 1 – 1) d = 2a + 2(n +1 -1)d

⇒ -a = 2nd – md

⇒ a = md- 2nd (i)


LHS:
a3m+1= a + (3m +1 -1)d
= md – 2nd + 3md
= 2d(2m - n)


RHS:
2a(m + n + 1) = 2[a +(m +n +1 -1)d]
= 2[md -2nd +md +nd]
= 2d(2m - n)


LHS = RHS

Hence, proved


Question 18.

If the nth term of the A.P.is same as the nth term of the A.P.find n.


Answer:

A=9, D=7-9=-2

a=15, d=-3


An=an


A+(n -1)D=a +(n -1)d


9 – (n -1)2=15 – (n -1)3


(n -1)(3 -2)=6


n -1=6


n=7



Question 19.

Find the 12th term from the end of the following arithmetic progressions:

(i)

(ii)

(iii)


Answer:

(i)


a=3, d=5 -3=2, an =201


an= a +(n -1)d


201 =3 +2n – 2


N =100


Now, we have to find 12th term from the last that means,


100th – 11=89th term


Then,


a89 =a + (89 – 1)d


=3 +88


=179


Hence, the 12th term from the end of the A.P. is 179.


(ii)


a=3, d= 8 -3 =5


an= 253


a + (n -1)d =253


3 + (n -1)5 =253


n=51


Now, we have to find 12th term from the last that means,


51th -11 = 40th term


Then,


a40 = a + (n -1)d


= 3 + 39(5)


=198


Hence, the 12th term from the end of the A.P. is 198


(iii)


a=1, d= 4 -1 =3


an= 88


a + (n -1)d =88


1 + (n -1)3 =88


n =30


Now, we have to find 12th term from the last that means,


30th -11 = 19th term


a19 = a + 18d


= 1 + 18(3)


=55


Hence, the 12th term from the end of the A.P. is 198.



Question 20.

The 4th term of an A.P. is three times the first and the 7th term exceeds twice the third term by 1. Find the first term and the common difference.


Answer:

Given:

a4= 3a1 …(i)


a7= 2a3 +1 …(ii)

We know an = a + (n-1) d

So,

a3 = a +2d

a4 = a +3d

a7 = a +6d

Put all the values of a1 and a4 in (i),

a + 3d = 3a

⇒ 3d = 3a - a

⇒ 3d = 2a

⇒ d=


Put all the values of a7 and a3 in (ii),


⇒ a + 6d= 2(a +2d) +1

⇒ a + 6d = 2a + 4d + 1

⇒ 6d - 4d = 2a - a +1

⇒ 2d = a +1


Put d = ,


⇒ 2() = a +1


= a + 1







⇒ a= 3


Now, difference, d=






⇒ d= 2


Hence, first term is 3 and the common difference is 2.


Question 21.

Find the second term and nth term of an A.P. whose 6th term is 12 and the 8th term is 22.


Answer:

a6= a + 5d

12= a + 5d …(i)


a8= a + 7d


22= a + 7d …(ii)


Subtracting (i) from (ii), we get,


10= 2d


d= 5


from (i)


12 = a + 5(5)


12= a + 25


a= -13


a2= a +d


=-13 +5


=-8


an= a + (n-1)d


= -13 + (n-1)5


=-18 +5n



Question 22.

How many numbers of two digit are divisible by 3?


Answer:

Two digit numbers divisible by 3 are 12, 15, 18,… ,99

Hence, a= 12, ,d=3


an= a + (n -1)d


99= 12(n -1)3


99= 9 = 3n


n=30


Hence, the total numbers of two digit divisible by 3 are 30.



Question 23.

An A.P. consists of 60 terms. If the first and the last terms be 7 and 125 respectively, find 32nd term.


Answer:

n =60, a=7, a60 =l =125

a60 = a + 59d


125 = 7 + 59d


d=


d=2


a32= a + 31d


= 7 + 31(2)


=69


Hence, 32th term is 69 in the given A.P.



Question 24.

The sum of 4th and 8th terms of an A.P. is 24 and the sum of the 6th and 10th terms is 34. Find the first term and the common difference of the A.P.?


Answer:

Given: a4 + a8 = 24 …(i)

a6 + a10 = 34 …(ii)


a4 = a + 3d


a8 =a + 7d


a6 =a + 5d


a10=a + 9d


Put the value of a4 and a8 in (i)


(a + 3d) + (a + 7d) = 24


a + 5d =12 …(iii)


Put the value of a6 and a10 in (ii)


(a +5d) + (a +9d) =34


a +7d =17 …(iv)


Subtracting (iii) from (iv), we get


2d = 5


d =


Putting the value of d in (iii), we get


a = 12 - =


Hence, first term is and common difference is



Question 25.

The first term of an A.P. is 5 and its 100th term is - 292. Find the 50th term of this A.P.


Answer:

Given, a = 5

a100= -292


5 – 99d = -292


d = = 3


Now, 50th term, a50 = a + (50 – 1) d


= 5 + (49) 3 = 5 + 147 = 152


Hence, 50th term of given A.P. is 152



Question 26.

Findfor the A.P.

(i)

(ii)


Answer:

(i)

a = -9, d = -14 + 9 = -5


a30= a + 29d


= -9 + 29(-5) = -154


a20= a + 19d


= -9 + 19(-5) = -104


a30 – a20= -154 – (-104)


= -154 + 104


= -50


(ii)


a = a, d = a + d – a = d


a30= a + 29d


a20= a+ 19d


a30 – a20 = a + 29d – a – 19d


= 10d



Question 27.

Write the expressionfor the A.P.

Hence, find the common difference of the A.P. for which

(i)11th terms is 5 and 13th term is 79.

(ii)

(iii) 20th term is 10 more than the 18th term.


Answer:

Let nth term an = a + (n – 1) d

= a + nd – d


kth term, ak = a + (k – 1) d


= a + kd – d


Now,


an - ak = (a + nd – d) – (a + kd – d)


= nd – kd = d (n – k)


(i) a11 = 5


a + 10d = 5 (i)


a13 = 79


a + 12d = 79 (ii)


By subtracting (i) from (ii), we get


2d = 74


d = 37


Hence, the common difference is 37


(ii)a10 = a + 9d (i)


a5 = a + 4d (ii)


a10 – a5 = a + 9d – a – 4d


200 = 5d


d = 40


Hence, common difference is 40


(iii) a20 = a + 19d (i)


a18 = a + 17d (ii)


Given that a20 = a18 + 10


a + 19d = a + 17d + 10


2d = 10


d = 5


Hence, common difference is 5



Question 28.

Find n if the given value of x is the nth term of the given A.P.

(i)

(ii)

(iii)

(iv)


Answer:

(i)

a = 25, d = 50 – 25 = 25


Last term, l = 1000


Number of terms, n = + 1


= + 1 = 40


Hence, the value of n is 40


(ii)


a = -1, d = -2


Last term, l = -151


Number of terms, n = + 1


= + 1 = 76


Hence, the value of n is 76


(iii)


a = , d =


Last term, l = 550


l = a + (n – 1) d


550 = + (n – 1)


550 =


1100 = 11n


n = 100


Hence, number of terms, n = 100


(iv)


a = 1, d = – 1 =


Last term, l =


a + (n – 1) d =


1 + (n – 1) =


= – 1 +


=


10n = 170


n = 17


Hence, value of n is 17



Question 29.

If an A.P. consists of n terms with first term a and nth term ls how that the sum of the mth term from the beginning and the mth term from the end is (a+l).


Answer:

Given, a = a

Last term, l = l


We have to prove that the sum of the mth term from the beginning and mth term from the end is (a + 1)


Now, mth term from the beginning,


am(b) = a + (m – 1) l


= a + ml – l (i)


Again mth term from the end


am(e) = l – (m – 1)l


= l – ml + l = 2l – ml (ii)


By adding (i) and (ii), we get


a + ml – l + 2l – ml = a + l


Hence, proved



Question 30.

Find the arithmetic progression whose third term is 16 and seventh term exceeds its fifth term by 12.


Answer:

Given, a3 = 16

a + 2d = 16 (i)


a5 = a + (5 – 1) d


= a + 4d


a7 = a + (7 -1)d


= a + 6d


Acc. To question,


a7 = 12 + a5 (ii)


Put the value of a5 and a7 in (ii), we get


a + 6d = 12 + a + 4d


2d = 12 + a – a


2d = 12


d = 6


From equation (i),


16 = a + 2(6)


16 = a + 12


a = 4


We have to find A.P.


a1 = 4


a2 = a + d = 4 + 6 = 10


a3 = a + 2d = 4 + 2(6) = 16


a4 = a + 3d = 4 + 3(6) = 22


Hence, the A.P. is 4 , 10, 16 , 22,…



Question 31.

The 7th term of an A.P. is 32 and its 13th term is 62. Find the A.P.


Answer:

a7 = 32

a + 6d = 32 (i)


a13 = a + 12d = 62 (ii)


By subtracting (i) from (ii), we get


6d = 30


d= 5


From (i), a = 32 – 6(5) = 2


We have to find A.P.


a1 = 2


a2 = a + d = 2 + 5 = 7


a3 = a + 2d = 2 + 2(5) = 12


a4 = a + 3d = 2 + 3(5) = 17


Hence, A.P. is 2, 7, 12, 17,…



Question 32.

Which term of the A.P.will be 84 more than its 13th term?


Answer:

a = 3, d = 7

a13 = 3 + (13 – 1)7


= 3 + 84 = 87


Now, nth term is 84 more than the 13th term


an = 84 + a13


= 84 + 87


= 171


Now we have to find n,


an = a + (n – 1) d


171 = 3 + (n – 1) 7


7n = 175


n = 25


Hence, 25th term of the given A,.P. is 84 more than its 13th term



Question 33.

Two arithmetic progressions have the same common difference. The difference between their 100th terms is 100, what is the difference between their 1000th terms?


Answer:

Two arithmetic progressions have the same common difference.

Let, common difference = d


First term of first A.P. = a


First term of second A.P. = a’


Given that difference between their 100th term is 100


100th term of first A.P., a100 = a + 99d


100th term of second A.P., a’100 = a’ + 99d


Acc. To question,


a100 – a’100 = 100


a + 99d – a’ – 99d = 100


a – a’ = 100 (i)


1000th term of first A.P., a1000 = a + 999d


1000th term of second A.P., a’1000 = a’ + 999d


Acc. To question,


a1000 – a’1000 = a + 999d – a’ – 999d


= a – a’


a1000 – a’1000 = 100 [from (i)]


Hence, difference between 1000th term of two A.P. is 100



Question 34.

For what value of n, the nth terms of the arithmetic progressions andare equal?


Answer:

Given, A.P. = 63, 65, 67,…

A.P.’ = 3, 10, 17,…


Let for A.P.,


First term, a = 63, d = 65 – 63 = 2


And nth term = an


an = 63 + (n – 1) 2


= 63 + 2n – 2


= 61 + 2n


For A.P.’


First term, a = 3, d = 10 – 3 = 7


nth term = an


Now nth term of both A.P. are equal


61 + 2n = -4 + 7n


7n – 2n = 61 + 4


n = 13


Hence, 13th term of both the A.P. are equal



Question 35.

How many multiples of 4 lie between 10 and 250?


Answer:

Let, multiple of 4 lies between 10 and 250

12, 16, 20, 24,…248


We know, an = a + (n – 1)d


First term, a = 12


C0mmon difference, d = 16 – 12 = 4


Last term, an = 248


an = a+ (n – 1)d


248 = 12 + (n – 1) 4


248 = 12 + 4n – 4


4n = 248 – 12 + 4


4n = 240


n = 60


Hence, multiple of 4 lies between 10 and 250 is 60



Question 36.

How many three digit numbers are divisible by 7?


Answer:

Let, three digit number divisible by 7 are

105, 112, 119,….994


Here, First term, a = 105


Common difference, d = 112 – 105 = 7


And last term, an = 994


an = a+ (n – 1) d


994 = 105 + (n – 1) 7


994 = 105 + 7n – 7


994 = 98 + 7n


7n = 896


n = 128


Hence, three digit number that are divisible by 7 are 128



Question 37.

Which term of the arithmetic progression will be72 more than its 41th term?


Answer:

a = 8, d = 6,

Last term = an


an= a + (n – 1) d


= 8 + (n – 1) 6


= 2 + 6n (i)


Let, 41st term, a41 = 8 + (41 – 1) 6


= 8 + 40 * 6 = 248


Now the term is 72 more than its 41st term


an = 72 + a41


= 72 + 248 = 320


Putting this value in (i), we get


320 = 2 + 6n


6n = 318


n = 53


Hence, 53rd term of the given A.P. is 72 more than its 41st term



Question 38.

Find the term of the arithmetic progression which is 39 more than its 36th term.


Answer:

First term, a = 9, d = 12 – 9 = 3

Let, last term be an


an = a + (n – 1) d


= 9 + (n – 1)3


= 9 + 3n – 3 = 6 + 3n (i)


Let, 36th term, a36 = a + 35d


= 9 + 35 (3) = 114


Now the term is 39 more than its 36th term


an = 39 + a36


= 39 + 114 = 153


Putting the value in (i), we get


153 = 6 + 3N


3n = 153 – 6


3n = 147


n = 49


Hence, 49th term of the given A.P. is 39 more than its 36th term



Question 39.

Find the 8th term from the end of the A.P.


Answer:

a = 7, d = 3

Last term, an = 184


an = a + (n – 1) d


184 = 7 + (n - 1) 3


184 = 7 + 3n – 3


180 = 3n


n = 60


Now we have to find last 8th term, it means = 60th – 7 = 53rd term


a53 = 7 + (53 – 1)3


= 7 + 52 * 3


= 7 + 156 = 163


Hence, 8th term from the end of given is 163



Question 40.

Find the 10th term from the end of the A.P. 8, 10, 12, ..., 126


Answer:

Given,
First term, a = 8
Common difference, d = 2

Let the number of terms be 'n'
We know, nth term of an AP is
an = a + (n – 1)d
where ‘a’ and ‘d’ are first term and common difference of AP respectively
Last term, an = 126

⇒ an = a + (n – 1) d

⇒ 126 = 8 + (n – 1) 2

⇒ 120 = 2n

⇒ n = 60


Now we have to find last 10th term, means = 60th term – 10 + 1 = 51th term

Now, a51 = 8 + (51 – 1)2

= 8 + 50(2)
= 8 + 100
= 108


Hence, 10th term from the last of given A.P. is 108


Question 41.

The sum of 4th and 8th terms of an A.P. is 24 and the sum of 6th and 10th terms is 44. Find the A.P.


Answer:

Given, a4 + a8 = 24 (i)

a6 + a10 = 44 (ii)


We know, an = a + (n – 1) d


4th term, a4 = a + 3d


6th term, a6 = a + 5d


8th term, a8 = a + 7d


10th term, a10 = a + 9d


Putting the value of a4 and a8 in (i), we get


a + 3d + a + 7d = 24


2a + 10d = 24 (iii)


Put the value of a6 and a10 in (ii), we get


a + 5d + a + 9d = 44


2a + 14d = 44 (iv)


By subtracting (iii) from (iv), we get


4d = 20


d = 5


Now putting value of d in (iii), we get


2a = 24 – 10(5)


a = -13


a1 = -13


a2 = a + d = -13 + 5 = - 8


a3 = a + 2d = -13 + 10 = -3


Hence, the A.P. is -13, -8, -3,…



Question 42.

Which term of the A.P. will be 120 more than its 21st term?


Answer:

a = 3, d = 15 – 3 = 12

Let last term be an


an = a + (n – 1) d


= 3 + (n – 1) 12


= 12n – 9 (i)


a21 = a + 20d


= 3 + 20(12) = 243


Now, the term is 120 more than the 21st term


an = 120 + a21


= 120 + 243


= 363


Putting this value in (i), we get


363 = 12n – 9


12n = 363 + 9


n = 31


Hence, 31st term of given A.P. is 120 more than its 21st term



Question 43.

The 17th term of an A.P. is 5 more than twice its 8th term. If the 11th term of the A.P. is 43, find the nth term.


Answer:

Given: The 17th term of an A.P. is 5 more than twice its 8th term and the 11th term of the A.P. is 43.

To find: the nth term.

Solution:

Given,

a17 = 5 + 2(a8) ......(i)

And

a11 = 43

As an = a + (n-1) d

For n=11,

a + 10d = 43

a = 43 - 10d ......(ii)


For n = 8,


a8 = a + 7d


For n=17,


a17 = a + 16d


Put the value of a from (ii)


⇒ a17= 43 – 10d + 16d


⇒ a17 = 43 + 6d


Putting the value of a8 and a17 in (i), we get


⇒ 43 + 6d = 5 + 2(a + 7d)


⇒ 43 + 6d = 5 + 2a + 14d


⇒ 43 – 5 = 2a + 14d – 6d


⇒ 38 = 2a + 8d


⇒ 38 = 2(43 – 10d) + 8d [from (ii)]


⇒ 38 = 86 – 20d + 8d


⇒ 38 = 86 – 12d


⇒ 12d = 86 – 38

⇒ 12d = 48


⇒ d = 4


From (ii), a = 43 – 10d


⇒ 43 – (10 × 4) = 43 – 40 = 3


We know, nth term of A.P., an = a + (n -1) d


an = 3 + (n – 1) 4


an = 3 + 4n – 4


an = 4n – 1


Hence, nth term is 4n – 1.


Question 44.

Find the number of all three digit natural numbers which are divisible by 9.


Answer:

Let the three digit numbers divisible by 9 are

108, 117, 126,… ,999


a=108, d=9, last term, l= 999


Number of terms, n = +1


= + 1


= +1


=100


Hence, the number of all three digit natural numbers which are divisible by 9 are 100



Question 45.

The 19th term of an A.P. is equal to three times its sixth term. If its 9th term is 19, find the A.P.


Answer:

Given: a9 =19

a + 8d= 19 (i)


Acc. to question,


a19= 3a6 (ii)


a19=a + 18d


a6=a + 5d


Putting the value of a19 and a6 in (ii)


a + 18d =3(a +5d)


3d= 2a


3d=2(19 -8d) (from (i))


19d=38


d=2


Now, putting the value of d in (i)


a=19 -8(2) =3


a1 = a=3


a2 =a +d=3 +2=5


a3 =a +2d=3 + 2(2)=7


Hence, the A.P. is



Question 46.

The 9th term of an A.P. is equal to 6 times its second term. If its 5th term is 22, find the A.P.


Answer:

Given: a5 = 22

a + 4d=22 (i)


Acc. to question,


a9 = 6a2 (ii)


a9= a +8d


a2=a+d


Putting the value of a9 and a2 in (ii)


a +8d= 6(a +d)


2d=5a


2d=5(22 -4d)


22d=110


d=5


Now, putting the value of d in (i)


a=22 -4(5)=2


a1 = a=2


a2 =a +d=2 +5=7


a3 =a +2d=2 + 2(5)=12


Hence, the A.P. is



Question 47.

The 24th term of an A.P. is twice its 10th term. Show that its 72nd term is 4 times its 15th term.


Answer:

Given: a24=2a10

a +23d= 2(a +9d)


5d=a (i)


To Prove:a72 =4a15


Proof: L.H.S. =a72


= a +71d


=5d +71d [from (i)]


=76d


R.H.S= 4a15


=4(a +14d)


=4a +56d


=4(5d) +56d [from (i)]


=20d +56d


=76d


Since, L.H.S=R.H.S.


Hence proved



Question 48.

Find the number of natural numbers between 101 and 999 which are divisible by both 2 and 5.


Answer:

We have to find the numbers between 101 and 999 which are divisible by both 2 and 5

That means the numbers divisible by 10 between 101 and 999.


Let, the required divisible numbers be,


110, 120, 130, …, 990


a=110, d=10, last term, l=990


Number of terms, n = +1


= + 1


= +1


=89


Hence, the number of natural numbers between 101 and 999 which are divisible by both 2 and 5 are 89.



Question 49.

If the seventh term of an A.P. is 1/9 and its ninth term is 1/7, find its (63)rd term.


Answer:

We know, nth term of an AP is
an = a + (n – 1)d
where ‘a’ and ‘d’ are first term and common difference of AP respectively
Given,

Subtracting equation [1] from [2], we get

Putting this value of d in equation [1], we get

Now, 63rd term = a + 62d

Hence Proved!


Question 50.

The sum of 5th and 9th terms of an AP is 30. If its 25th term is three times its 8th term, find the AP.


Answer:

a5 +a9=30

(a +4d) + (a +8d)=30


2a +12d=30


a + 6d=15 (i)


Acc. To question,


a25 =3a8


a+ 24d = 3(a +7d)


3d=2a


3d=2(15 -6d) [from(i)]


15d=30


d=2


Putting the value of d in (i),


a= 15 – 6(2)=3


a1 = a=3


a2 =a +d=3 +2=5


a3 =a +2d=3 + 2(2)=7


Hence, the A.P. is



Question 51.

Find where 0 (zero) is a term of the A.P.


Answer:

a=40, d=37 -40= -3

Let, an =0


a + (n -1) d =0


40 + (n -1) (-3) =0


-3n = -43


n=43/3


Since, ‘n’ can’t be a fraction


Hence, the answer is no



Question 52.

Find the middle term of the A.P..


Answer:

a=213, l=37

Middle term of A.P. =


=


=


=125



Question 53.

If the 5th term of an A.P. is 31 and 25th term is 140 more than the 5th term, find the A.P.


Answer:

a5=31

a +4d =31 (i)


a25= 140 + a5


a + 24d= 140 + 31


31 -4d +24d=171 [ from (i) ]


20d=140


d=7


Putting the value of d in (i)


a= 31 – 4(7) = 3


a1 = a=3


a2 =a +d=3 +7=10


a3 =a +2d=3 + 2(7) =17


Hence, the A.P. is 3, 10, 17,…




Exercise 9.4
Question 1.

The sum of three terms of an A.P. is 21 and the product of the first and the third terms exceeds these term by 6, find three terms.


Answer:

Let the three terms be a - d, a, a + d

a – d + a + a – d = 21 (given)

3a = 21

a = 7

According to question,

(a – d) (a + d) = a + 6

a2 – d2 = a + 6

72 – d2 = 7 + 6

49 – d2 = 13

d2 = 36

d = 6

Therefore, a – d = 7 – 6 = 1

a + d = 7 + 6 = 13

Thus, the three terms are 1, 7 and 13.


Question 2.

Three numbers are in A.P. If the sum of these numbers be 27 and the product 648, find the numbers.


Answer:

Let the three numbers be a – d, a, a + d

a – d + a + a + d = 27


3a = 27


a = 9


According to question,


(a – d) (a) (a + d) = 648


(a2 – d2) (a) = 648


(92 – d2) 9 = 648


(81 – d2) = 72


d2 = 9


d = 3


Therefore, a – d = 9 – 3 = 6


A + d = 9 + 3 = 12



Question 3.

Find the four numbers in A.P. whose sum is 50 and in which the greatest number is 4 times the least.


Answer:

Let the four numbers be (a – 3d), (a – d), (a + d) and (a + 3d)

a – 3d + a – d + a + d + a + 3d = 50


4a = 50


a =


According to question,


(a + 3d) = 4(a – 3d)


a + 3d = 4a – 12d


3a = 15d


5d =


d =


Therefore,


a – 3d = - = 5


a – d = - = 10


a + d = + = 15


a + 3d = + = 20



Question 4.

The angles of a quadrilateral are in A.P. whose common difference is 10°. Find the angles.


Answer:

Let the angles be (a – 3d), (a – d), (a + d) and (a + 3d)

a – 3d + a – d + a + d + a + 3d = 360o (sum of angles of quadrilateral)

4a = 360o

a = 90o

According to question,

(a + d) – (a – d) = 10o

2d = 10o

d = 5o

Therefore, (a – 3d) = 90o – 15o = 75o

(a – d) = 90o – 5o = 85o

(a + d) = 90o + 5o = 95o

(a + 3d) = 90o + 15o = 105o


Question 5.

The sum of three numbers in A.P. is 12 and the sum of their cube is 288. Find the numbers.


Answer:

Let the numbers be (a – d), a, (a+ d)

A – d + a + a+ d = 12


3a = 12


a = 4


According to question,


(a – d)3 + a3 + (a + d)3 = 288


a3 – d3 – 3ad (a – d) + a3 + a3 + d3 + 3ad (a + d) = 288


3a3 – 3a2d + 3ad2 + 3a2d + 3ad2 = 288


3(4)3 + 6(4) d2 = 288


24d2 = 96


d2 = 4


d = �4


Therefore, when a = 4 and d = 2


a – d = 4 – 2 = 2


a + d = 4 + 2 = 6


When a = 4 and d = -2


a – d = 4 + 2 = 6


a + d = 4 – 2 = 2



Question 6.

Find the value of x for which (8x+4), (6x-2), (2x+7) are in A.P.


Answer:

We know that, if three numbers are a,b,c are in AP then, 2b = a + c

According to question, (8x+4), (6x-2), (2x+7) are in AP.

Hence,

2 (6x – 2) = 8x + 4 + 2x + 7

12x – 4 = 10x + 11

2x = 15

x = 15/2 = 7.5


Question 7.

If x+1, 3x and 4x+2 are in A.P, find the value of x.


Answer:

Given terms: x+1, 3x and 4x+2

Since, the given terms are in A.P.

Therefore, their common difference will be same

3x – (x + 1) = (4x + 2) – 3x

3x – x - 1 = 4x + 2 – 3x

2x - 1 = x + 2

x = 3


Question 8.

Show that (a–b)2, (a2+b2) and (a + b)2 are in A.P.


Answer:

The terms given below are : (a–b)2, (a2+b2) and (a + b)2

Common difference, d1 = a2 + b2 – (a – b)2

d1 = a2 + b2 – (a2 + b2 - 2ab)

d1 = a2 + b2 – a2 - b2 + 2ab

d1= 2ab

Common difference, d2 = (a + b)2 – (a2 + b2)

d2 = a2 + b2 + 2ab – a2 –b2

d2 = 2ab

Since, d1 = d2 i.e. the common difference is same.

Therefore, the given terms are in A.P.



Exercise 9.5
Question 1.

Find the sum of the following arithmetic progressions:

(i)to 10 terms

(ii)to 12 terms

(iii)to 25 terms

(iv)to 12 terms

(v)to 22 terms

(vi)to n terms

(vii)to n terms

(viii)to 36 terms


Answer:

(i)to 10 terms


Sn = [2a + (n – 1) d]


S10 = [100 + 9(-4)]


= 5 [100 – 36]


= 5(64) = 320


(ii)to 12 terms


Sn = [2a + (n – 1) d]


= [2 + (12 – 1) 2]


= 6 (2 + 22)


= 144


(iii)to 25 terms


Sn = [2a + (n – 1) d]


= [2 (3) + 24()]


= [6 + 36]


= 525


(iv)to 12 terms


Sn = [2a + (n – 1) d]


= [82 - 55]


= 6 [27] = 162


(v)to 22 terms


Sn = [2a + (n – 1) d]


= [2(a + b) + (21) (-2b)]


= 11 [2a + 2b – 42b]


= 11 [2a – 40b]


= 22a – 440b


(vi)to n terms


Sn = [2a + (n – 1) d]


= [2(x – y)2 + (n – 1) (2xy)]


= (2) [(x – y)2 + (n – 1) xy]


= n [(x – y)2 + (n – 1) xy]


(vii)to n terms


Sn = [2a + (n – 1) d]


= [2() + (n – 1) xy]


= {2(x – y) + (n – 1) (2x – y)}


= {n (2x – y) – y}


(viii)to 36 terms


Sn = [2a + (n – 1) d]


= [-52 + (35) 2]


= 18 [-52 + 70]


= 18 [18] = 324



Question 2.

Find the sum on n term of the A.P.


Answer:

Sn = [2a + (n – 1) d]

= [10 + (n – 1) -3]


= [13 – 3n]



Question 3.

Find the sum of n terms of an A.P. whose nth term is given by


Answer:

Put n = 1

a1 = 5 – 6(1) = 5 – 6 = -1


Put n = 2


a2= 5 – 6(2) = 5 – 12 = -7


a = -1, d = -7 + 1 = -6


Sn = [2a + (n – 1) d]


= [2(-1) + (n -1) (-6)]


= [-1 + 3n + 3]


= n [2 -3n]



Question 4.

If the n sum of a certain number of terms starting from first term of an A.P. is is 116. Find the last term.


Answer:

a = 25, d = -3

Sn = [2a + (n – 1) d]


116 = [50 – 3n +3]


116 = [53 – 3n]


232 = 53n – 3n2


3n2 – 53n + 232 = 0


n = 8 or n = , which isn’t possible as n must be a natural number.


Therefore, n = 8


[a + l] = 116


[25 + l] = 116



l = 4


Hence, the last term is 4



Question 5.

How many terms of the sequence should be taken so that their sum is zero?


Answer:

Sn = [2a + (n – 1) d]

0 = [2(18) + (n – 1) (-2)]


38n – 2n2 = 0


n = 0 (which is not possible)


n = 19


Therefore, n = 19 terms



Question 6.

How many terms are there in the A.P. whose first and fifth terms are 14 and 2 respectively and the sum of the terms is 40?


Answer:

a5= a + (n -1) d

2 = -14 + 4d


16 = 4d


d = 4


Sn = [2a + (n – 1) d]


40 = [2(-14) + (n -1) (4)]


40 = 12n2 – 16n


2n2 – 16n -40 = 0


n2 – 8n -20 = 0


n2 – 10n + 2n – 20 = 0


(n – 10) (n + 2) = 0


n = 10 and n = -2 (not possible)



Question 7.

How many terms of the A.P.must be taken so that their sum is 636?


Answer:

Sn = [2a + (n – 1) d]

636 = [2(9) + (n – 1) 8]


636 = n [9 + 4n -4]


636 = 5n + 4n2


4n2 + 5n – 636 = 0


n = 12 or n = (not possible)



Question 8.

How many terms of the A.P. must be taken so that their sum is 693?


Answer:

Sn = [2a + (n – 1) d]

693 = [2(63) + (n – 1) (-3)]


693 = [126 -3n + 3]


462 = n [43 – n]


n2 – 43n + 462 = 0


n = 22 or n = 21



Question 9.

The first and the last terms of an A.P. are 17 and 350 respectively. If the common difference is 9, how many terms are there and what is their sum?


Answer:

a = 17, l = 350, d = 9

Number of terms, n = + 1


= + 1


= = = 38


Sn= (a + l)


= (350 + 17)


= 19 (367) = 6973



Question 10.

The third term of an A.P. is 7 and the seventh term exceeds three times the third term by 2. Find the first term, the common difference and the sum of first 20 terms.


Answer:

We know, nth term of an AP is
an = a + (n – 1)d
where ‘a’ and ‘d’ are first term and common difference of AP respectively
Given, third term of an A.P. is 7
a3 = 7
⇒ a + 2d = 7 (i)
the seventh term exceeds three times the third term by 2

⇒ a7 = 3a3 + 2
⇒ a + 6d = 3(7) + 2
⇒ 7 – 2d +6d = 21 + 2
⇒ 4d = 16
⇒ d = 4


From (i),
a = 7 – 2(4)

= -1


Also, we know sum of first 'n' terms of an AP is

Sum of first 20 terms is

= 10[-2 + 76]
= 740


Question 11.

The first term of an A.P. is 2 and the last term is 50. The sum of all these terms is 442. Find the common difference.


Answer:

a = 2, l = 50

Sn= [a + l]


442 = [50 +2]


n = = 17


n = + 1


17 = + 1


16d = 48


d = 3



Question 12.

If 12th term of an A.P. is 13 and the sum of the first four terms is 24, what is the sum of first 10 terms?


Answer:

a12= -13

a + (12 – 1) d = -13


a + 11d = -13


S4 = [2a + (4 – 1) d]


24 = 2 [2a + 3d]


12 = 2a + 3d


12 = 2 (-11d – 13) + 3d


12 = -22d – 26 + 3d


38 = -19d


d = -2


a = -13 + 11(2)


= -13 + 22


= 9


S10= [2(9) + (10 – 1) (-2)]


= 5 [18 – 18]


= 0



Question 13.

Find the sum of first 22 terms of an A.P. in which d = 22 and a22 = 149.


Answer:

a22= a + 21d

149 = a + 21(22)


a = 149 – 462


a = -313


S22= [2a + (22 – 1) d]


= 11 [-626 + 462]


= 11 (-164)


= -1804



Question 14.

Find the sum of all natural numbers between a and 100, which are divisible by 3.


Answer:

a = 3, l = 99, n = 33

S33= (a + l)


= (3 + 99)


= (102)


= 33 (51) = 1683



Question 15.

Find the sum of first n odd natural numbers.


Answer:

a = 1, d = 2

Sum of first n odd numbers, Sn = [2a + (n – 1) d]


= [2(1) + (n – 1) 2]


= n [1 + n – 1]


= n2



Question 16.

Find the sum of all odd numbers between

(i) 0 and 50

(ii) 100 and 200


Answer:

(i) 0 and 50


a = 1, l = 49, n = 25


S25= [a + l]


= [1 + 49]


= (50) = 625


(ii) 100 and 200


a = 101, l = 199, n = 50


S50= [a + l]


= 25 (101 + 199)


= 25 (300) = 7500



Question 17.

Show that the sum of all odd integers between 1 and 1000 which are divisible by 3 is 83667.


Answer:

a = 3, l = 999, n = = 167

Sn= (a + l)


= (3 + 999)


= (1002)


= 167 (501) = 83667



Question 18.

Find the sum of all integers between 84 and 719, which are multiples of 5.


Answer:

To find: the sum of all integers between 84 and 719, which are multiples of 5.

Solution:


The smallest and largest digits between 84 and 719 which are divisible by 5 is 85 and 715.

So the sequence will be 85,90,........... 715.

a = 85, d = 5, l = 715


From the formula l=a+(n-1)d

The number of terms are:

n = + 1


n= + 1


n=


n= 127


So there are 127 terms between 84 and 719.


Now to calculate the sum of 127 terms use the formula:


⇒ S127= (a + l)


⇒ S127= (85 + 715)


⇒ S127= (800) = 50800

So the sum of the terms between 84 and 719 is 50800.


Question 19.

Find the sum of all integers between 50 and 500, which are multiples of 7.


Answer:

a = 56, d = 7, l = 497

n = + 1


= + 1


= + 1 =


= 64


S64= (a + l)


= 32 (56 + 497)


= 32 (553) = 17696



Question 20.

Find the sum of all even integers between 101 and 999.


Answer:

a = 102, l = 998

n = = 449


S449= (a + l)


= (102 + 998)


= = 246950



Question 21.

Find the sum of all integers between 100 and 550, which are multiples of 9.


Answer:

a = 108

L = 549


D = 9


549 = 108 + (n - 1)9


549 = 99 + 9n


9n = 450


n = 50


Sn= [216 + 49 (9)]


= 16425



Question 22.

In an A.P. if the first term is 22, the common difference is – 4 and the sum to n terms is 64, find n.


Answer:

a = 22, d = -4

Sn = 64


64 = (2a + (n – 1) d)


n [44 – (n – 1)4] = 128


n (44 – 4n + 4) = 128


48n – 4n2 = 128


n2 – 12n + 32 = 0


n2 – 8n – 4n + 32 = 0


(n – 8) (n – 4) = 0


n = 8 or n = 4



Question 23.

In an A.P. If the 5th and 12th terms are 30 and 65 respectively, what is the sum of first 20 terms?


Answer:

According to question,

a5= 30


a + 4d = 30


a = 30 – 4d (i)


a12= 65


a + 11d = 65


30 – 4d + 11d = 65 [from (i)]


7d = 35


d = 5


Put d in (i)


a = 30 – 4(5)


= 10


S20= [2(a) + (20 – 1) d]


= 10 [2(10) + 19(5)]


= 10 {20 + 95}


= 1150



Question 24.

Find the sum of the first

(i) 11 terms of the A.P :

(ii) 13 terms of the A.P :

(iii) 51 terms of the A.P. whose second term is 2 and fourth term is 8.


Answer:

(i) 11 terms of the A.P :


a = 2, d = 6 – 2 = 4


S11= [2(a) + 10d]


= [2(2) + 10(4)]


= 11 [2 + 20]


= 242


(ii) 13 terms of the A.P :


a = -6, d = 0 + 6 = 6


S13= [2(a) + 12d]


= 13 [-6 + 6(6)]


= 13 [-6 + 36]


= 13 (30) = 390


(iii) 51 terms of the A.P. whose second term is 2 and fourth term is 8.


a2= 2


a + d = 2 (i)


a4 = 8


a + 3d = 8


2 – d + 3d = 8


2 + 2d = 8


d = 3


a = -1


S51= [2(a) + 50d]


= 51 [-1 + 25(3)]


= 51 (74)


= 3774



Question 25.

Find the sum of

(i) The first 15 multiples of 8

(ii) The first 40 positive integers divisible by (a) 3 (b) 5 (c) 6.

(iii) All 3 – digit natural numbers which are divisible by 13.

(iv) All 3 – digit natural numbers, which are multiples of 11.


Answer:

(i) The first 15 multiples of 8


a = 8, d = 8


S15= [2(8) + 14(8)]

= 15[8 + 56]

= 15[64]

= 960


(ii) The first 40 positive integers divisible by (a) 3 (b) 5 (c) 6.


(a) a = 3, d = 3


S40= [2(a) + 39(d)]


= 20 [2(3) + 39(3)]


= 60 (41) = 2460


(b) a = 6, d = 5


S40= [2(5) + 39(5)]


= 100 [41]


= 4100


(iii) All 3 – digit natural numbers which are divisible by 13.


a = 6, d = 6


S40= [2(6) + 39(6)]


= 120 (41)

= 4920


(iv) All 3 – digit natural numbers, which are multiples of 11.


a = 110, d = 11, l = 990


n = + 1


= + 1


=

=


= 81


S81= [2(110) + 80(11)]


= 8910 [1 + 4]


= 44550


Question 26.

Find the sum :

(i)

(ii)

(iii)

(iv)

(v)

(vi)

(vii)

(viii)


Answer:

(i)


a = 2, d = 4 – 2 = 2, l = 200


n = + 1


= + 1 = 100


S100= [2(2) + 99(2)]


= 100 [101] = 10100


(ii)


a = 3, d = 11 – 3 = 8, l = 803


Sn= + 1 = + 1


= 101


S101= [2(3) + 100(8)]


= 101 (403) = 40703


(iii)


a = -5, d = -8 + 5 = -3, l = -230


n = + 1


= = 76


Sn= 38(5) [-2 – 45]


= 190 (-47) = -8930


(iv)


a = 1, d = 3 – 1 = 2, l = 199


n = + 1


= 100


S100= [2(1) + 99(2)]


= 100 (100) = 10000


(v)


a = 7, d = - 7 =


Last term, an = 84


an = a + (n – 1)d


84 = 7 + (n – 1)


84 =


84 * 2 = 7 + 7n


168 = 7 + 7n


7n = 168 – 7


7n = 161


n = 23


Sum of nth term, Sn = [2a + (n – 1)d]


S23 = [2(7) + (23 – 1)]


= [14 + 22 * ]


= [14 + 77]


= * 91 =


Hence, sum of given A.P. is


(vi)


a = 34, d = 32 – 34 = -2


Last term, an = 10


an = a + (n – 1)d


10 = 34 + (n – 1) (-2)


10 = 34 – 2n + 2


2n = 34 – 10 + 2


2n = 26


n = 13


Sum of n terms,


Sn = [2a + (n – 1) d]


S13 = [2(34) + (13 – 1) (-2)]


= [68 + 12(-2)]


= [68 – 24]


= * 44 = 286


Hence, Sum of given A.P. is 286


(vii)


a = 25, d = 3


Last term, an = 100


An = a + (n -1) d


100 = 25 (n – 1)3


75 = 3n – 3


78 = 3n


n = 26


Sum of n terms, Sn = [2a + (n – 1) d]


= [2(25) + (26 – 1) 3]


= 13 [50 + 25 * 3]


= 13 * 125 = 1625


Hence, Sum of given A.P. is 1625


(viii)


a = 18, d = - 18 =


Last term, an =


an = a+ (n – 1)d


= 18 + (n – 1) ()


-99 = 36 + 5 – 5n


-140 = -5n


n = 28


S28= [a + l]


= 14 (18 - )


= 7 (36 – 99)


= 7 (-63) = -441



Question 27.

Find the sum of the first 15 terms of each of the following sequences having nth term as

(i)

(ii)

(iii)

(iv)


Answer:

(i)


Put n = 1


a1 = 3 + 4(1) = 7


Put n = 15


a15 = 3 + 4(15) = 63 = l


Sum of 15 terms, S15 = [a + l]


= [7 + 63] = 525


(ii)


Put n = 1


b1 = 5 + 2(1) = 7


Put n = 15


b15 = 5 + 2(15) = 35 = l


Sum of 15 terms, S15= [a + l]


= [7 + 35] = 315


(iii)


Put n = 1


x1 = 6 – 1 = 5


Put n = 15


x15 = 6 – 15 = -9


Sum of 15 terms, S15= [a + l]


= [5 - 9] = -30


(iv)


Put n = 1


y1= 9 – 5(1) = 4


Put n = 15


y15= 9 – 5(15) = -66


Sum of 15 terms, S15= [a + l]


= [4 - 66] = -465



Question 28.

Find the sum of first 20 terms of these sequence whose nth term is .


Answer:

Given, an= An + B

Put n = 1, a1= A + B


Put n = 20, a20= 20A + B


S20= [a + l]


= 10 [A + B + 20A + B]


= 10 [21A + 2B]


= 210A + 20B


Question 29.

Find the sum of first 25 terms of an A.P. whose nth term is given by .


Answer:

Given, nth term, an = 2 – 3n

Put n = 1, a1 = 2 – 3(1) = -1


Put n = 25, a15= 2 – 3(15) = -43


Therefore, S25 = (-1 – 43)


= (-44) = -925



Question 30.

Find the sum of the first 25 terms of an A.P. whose nth term is given by .


Answer:

Given, an= 7 - 3n

Put n = 1


a1= 7 – 3(1) = 4


Put n = 25


a25= 7 – 3(25) = -68 = l


S25= [4 – 68]


= 25 [-32] = -800



Question 31.

Find the sum of the first 25 terms of an A.P. whose second and third terms are 14 and 18 respectively.


Answer:

a2= 14

a + d = 14 (i)


a3= 18


a + 2d = 18 (ii)


Subtracting (i) from (ii), we get


d = 4


Putting the value of d in (i), we get


a = 14 – 4 = 10


S25= [2(a) + 24(d)]


= 25 [58]


= 1450



Question 32.

If the sum of 7 terms of an A.P. is 49 and that of 17 term is 289, find the sum of n terms.


Answer:

S7= 49

[2a + 6d] = 49


a + 3d = 7 (i)


S17= 289


[2a + 16d] = 289


a + 8d = 17 (ii)


Subtract (i) from (ii), we get


5d = 10


d = 2


Put d = 2 in (i), we get


a = 7 – 6 = 1


Sn= [2(1) + (n – 1)2]


= n [1 + n – 1]


= n2



Question 33.

The first term of an A.P. is 5, the last term is 45 and the sum is 400. Find the number of terms and the common difference.


Answer:

a = 5, l = 45

Sn= 400


[5 + 45] = 400


[50] = 400


n = 16


16th term is 45


a16= 45


5 + 15d = 45


15d = 40


d =



Question 34.

In an A.P., the sum of first n terms is . Find its 25th term.


Answer:

Given,

Let an be the nth term of the A.P.


an= Sn – Sn – 1


= 3n2/2 + 13n/2 – 3(n – 1)2/2 – 13(n – 1)/2


= {n2 – (n – 1)2} + {n – (n – 1)}


= 3n - +


= 3n + = 3n + 5


Put n = 25


a25= 3(25) + 5


= 75 + 5 = 80


Therefore, 25th term is a25 = 80


Question 35.

Let there be an A.P. with first term ‘a’, common difference, d. Ifdenotes its nth term andthe sum of first n terms, find.

(i)

(ii)

(iii)

(iv)

(v)

(vi)


Answer:

(i)


Given, an= 50


a + (n – 1) d = 50


5 + (n – 1) 3 = 50


(n – 1)3 = 45


(n -1) = 15


n = 16


Sn= [a + l]


= 8 [5 + 50] = 8 [55] = 440


(ii)


an= 4, d = 2, Sn= -14


a + (n – 1) 2 = 4


a + 2n = 6, and


[2a + (n – 1)2] = -14


n [ 2a + 2n – 2] = -14, or


[a + an] = -14


[a + 4] = -14


n [6 -2n +4] = -28


n [10 -2n] = -28


2n2 – 10n – 28 = 0


2(n2 – 5n – 14) = 0


(n + 2) (n – 7) = 0


n = -2, n = 7


Therefore, n = -2 is not a natural number. So, n = 7


(iii)


Given, a = 3, n = 8, Sn= 192


Sn = {2a + (n – 1) d]


192 * 2 = 8 [6 + (8 – 1)d]


= 6 + 7d


48 = 6 + 7d


7d = 42


d = 6


(iv)


an= 28, Sn = 144, n = 9


Sn= [a + l]


144 = [a + 28]


= a = 28


a + 28 = 32


a = 4


(v)


Given, a = 8, an= 62 and Sn= 210


Sn= [a + l]


210 = [8 + 62]


210 * 2 = n (70)


n = = 6


a + (n – 1) d = 62


8 + (6 – 1) d = 62


5d = 54


d = 10.8


(vi)


Given, a = 2, d = 8 and Sn = 90


90 = [4 + (n – 1)d] {Therefore, Sn= [2a + (n – 1)d]}


180 = n [4 + 8n – 8]


8n2 – 4n – 180 = 0


4 (2n2 – n – 45) = 0


2n2 – n – 45 = 0


(2n + 1) (n – 5) = 0


Therefore, n = is not a natural number


n = 5


an= a + (n – 1)d


= 2 + 4(8) = 32



Question 36.

Aman saved Rs. 16500 in ten years. In each year after the first he saved Rs.100 more than he did in the preceding year. How much did he save in the first year?


Answer:

Given,

Man saved in 10 years, S10= 16500


In each year after first he saved, d = 100


We know, sum of n terms, Sn= [2a + (n – 1) d]


For 10 years


S10= [2a + (10 – 1) 100]


16500 = 5 [2a + 9 * 100]


= 2a + 900


3300 = 2a + 900


2a = 3300 – 900


2a = 2400


a = 1200


Hence, Rs. 1200 saved by him in first year



Question 37.

Aman saved Rs.32 during the first year, Rs.36 in the second year and in this way he increases his savings by Rs.4 every year. Find in what time his saving will be Rs.200.


Answer:

Given, A man saved in first year, a = 32

A man saved in second year, a2 = 36


Increase saving, d = 4


In n years his saving will be 200, Sn = 200


We know,


Sn = [2a + (n – 1) d]


200 = [2(32) + (n – 1) 4]


400 = n [64 + 4n – 4]


400 = n [60 + 4n]


400 = 4n [15 + n]


100 = 15n + n2


n2 + 15n – 100 = 0


n2 + 20n – 5n – 100 = 0


n (n + 20) – 5 (n + 20) = 0


(n – 5) (n + 20) = 0


Here, n – 5 = 0, n = 5


n + 20 = 0, n = -20


The term can never be negative. So, we consider n = 5


Hence, in 5 years his saving will be Rs. 200



Question 38.

Aman arrange stop ay off a debt of Rs.3600 by 40 annual installments which for man arithmetic series. When 30 of the installments are paid, he dies leaving one - third of the debt unpaid, find the value of the first installment.


Answer:

Given, Total amount of payable in 40 annual installments, S40 = 3600

After 30 installment he died and leaving of the debt unpaid


Which means, total in 30 installment, S30= of the debt


S30= * 3600 = 2400


We know sum of n terms, Sn = [2a + (n – 1) d]


For 30 installments, S30 = [2a + (30 – 1) d]


2400 = 15 [2a + 29d]


160 = 2a + 29d


2a = 160 – 29d


a = (i)


For 40 installments, S40 = [2a + (n – 1) d]


3600 = [2a + (40 – 1) d]


180 = 2a + 39d


2a = 180 – 39d


a = (ii)


From (i) and (ii), we get


=


160 – 29d = 180 – 39d


39d – 29d = 180 – 160


10d = 20


d = 2


Putting the value of d in (i), we get


a =


= = = 51


Hence, value of first installment is 51



Question 39.

There are 25 trees at equal distances of 5 metres in a line with a well, the distance of the well from the nearest tree being 10 metres. A gardener waters all the trees separately starting from the well and here turns to the well after watering each tree to get water for the next. Find the total distance the gardener will cover in order to water all the trees.


Answer:

Total numbers of trees = 25

Distance between trees = 5


Total distance between well and first tree is = 10 m


Now, gardener return back to well after watering each tree then,


Distance between well and second tree is = 10 + 10 + 5 = 25


Distance between well and third tree = 25 + 5 + 5 = 35


Distance between well and fourth tree = 35 + 5 + 5 = 45


Distance between well and last tree without return back = 10 + 24(5) = 10 + 120 = 130


So, common difference = 35 – 25 = 10


Now, Total distance covered by 24 trees,


We knew, Sn= [2a + (n – 1) d]


= [2(25) + (24 – 1) 10]


= 12 [50 + 23 * 10]


= 12 [50 + 230]


= 12 * 280


= 3360


Now, distance covered by 25 trees = 10 + 3360 = 3370


Total distance covered by gardener with return back = 3370 + 130 = 3500 m


Hence, total distance will covered by gardener is 3500.



Question 40.

A man is employed to count Rs.10710. He counts at there of Rs.180 per minute for half an hour. After this, he counts at the rate of Rs.3 less every minute than the preceding minute. Find the time taken by him to count the entire amount.


Answer:

Total amount for counting = Rs. 10710

In 1 min he counts =Rs. 180

In half an hour (30 min) he count = Rs.180×30 =Rs.5400

Amount left count after half an hour, Sn= Rs.10710 –Rs. 5400 = 5310

Now, in 31st min he count 3 less than preceding minute = Rs.180 – Rs.3 = Rs.177

In 32nd min he count 3 less than preceding minute = Rs. 177 –Rs. 3 =Rs. 174

Then, Arithmetic progression formed is 177, 174,…

Here, difference between minutes = 174 – 177 = -3

We know, sum of n terms, Sn= (n/2) [2a + (n – 1) d]

5310 = (n/2) [2(177) + (n – 1) -3]

5310 × 2 = n [354 – 3n + 3]

10620 = 354n – 3n2 + 3n

10620 = 357n – 3n2

⇒ 3n2 – 357n + 10620 = 0

On taking 3 common from the complete equation, we get,

⇒ 3 (n2 – 119n + 3540) = 0

⇒ n2 – 119n + 3540 = 0

Now, from factoring by splitting the middle term method, we get,

⇒ n2 – 60n – 59n + 3540 = 0

⇒ n (n – 60) – 59 (n -60) = 0

⇒ (n – 59) (n – 60) = 0

Therefore, either n = 59 or n = 60

We will use 59 as these are minutes

So The total time to calculate whole amount is 59 + 30 = 89 min
= 1 hr 29 min


Question 41.

A piece of equipment cost a certain factory Rs.600,000. If it depreciates in value, 15% the first, 13.5% the next year, 12% the third year, and soon. What will be its value at the end of 10 years, all percentages applying to the original cost?


Answer:

Given: A piece of equipment cost a certain factory Rs.600,000. If it depreciates in value, 15% the first, 13.5% the next year, 12% the third year, and soon.

To find: The value at the end of 10 years if all percentages applying to the original cost.

Solution:

Cost of equipment = 6, 00, 000

In 1 year the value depreciate by 15%


⇒ The value of the equipment after first year= 6, 00, 000 * = 90, 000


In 2 year depreciate by 13.5%



⇒ The value of the equipment after second year= 6, 00, 000 * = 81, 000


In 3 year depreciate by 12%



⇒ The value of the equipment after third year= 6, 00, 000 * = 72, 000


Now A.P. is 90000, 81000, 72000,…


So, common difference = 81000 – 90000 = -9, 000


We have to find total depreciation for 10 years,


Since the value of depreciation is constant in any consecutive years i.e its value is -9000.



So to find the depreciation after ten years we will use the formula:


Sn= [2a + (n – 1) d]


S10= [2(90, 000) + (10 – 1) (-9000)]


S10= 5 [180000 + 9 * (-9000)]


S10= 5 [180000 – 81000]


S10= 5 [99000] = 495000


Hence,


cost of equipment at the end of 10 years = original cost – depreciation


= 6, 00, 000 – 4, 95, 000


= Rs. 1, 05, 000


Question 42.

A sum of Rs.700 is to be used to gives even cash prizes to students of a school for their overall academic performance. If each prize is Rs.20 less than its preceding prize, find the value of each prize.


Answer:

Amount of money = 700

Total number of prize = 7


Each prize is Rs. 20 less than its preceding prize


Let, Prize of first prize = a


Prize of second prize = a – 20


Prize of third prize = a – 20 – 20 = a – 40


Here, A.P. is a, a – 20, a – 40,…


So, common difference, d = a – 20 – a = -20


We know, Sn = [2a + (n – 1) d]


700 = [2a + (7 – 1) -20]


= 2a + 6 (-20)



2a = 320


a = 160


So, value of first prize = 160


Value of second prize, 160 – 20 = 140


Value of third prize, 140 – 20 = 120


Value of fourth prize, 120 – 20 = 100


Value of fifth prize, 100 – 20 =80


Value of sixth prize, 80 – 20 = 60


And value of seventh prize, 60 – 20 = 40


Hence, value of prizes, Rs 160, Rs 140, Rs 120, Rs 100, Rs 80, Rs 60, Rs 40.



Question 43.

In an A.P. the first term is 8, nth term is 33 and the sum to first n terms is 123. Find n and d, the common differences.


Answer:

Given, First term, a = 8

Nth term, an = 33 = l


And sum of n terms, Sn = [a + l]


123 = [8 + 33]


123 * 2 = n * 41


n = = = 6


We know,


an= a + (n – 1) d


33 = 8 + (6 – 1) d


33 – 8 = 5d


25 = 5d


d = 5


Hence, number of terms n = 6 and common difference, d = 5



Question 44.

In an A.P., the first term is 22, nth term is -11 and the sum to first n terms is 66. Find n and d, the common difference.


Answer:

Given, First term, a = 22

nth term, an= -11 = l


and sum of n terms, Sn= 66


We know sum of n terms, Sn= [a + l]


66 = [22 – 11]


66 * 2 = n * 11


n = = 12


we know, an= a + (n – 1) d


-11 = 22 + (12 – 1) d


-11 – 22 = 11d


-33 = 11d


d = -3


Hence, number of terms, n = 12 and common difference, d = -3



Question 45.

If the sum of the first n terms of an A.P. is 4n – n2, which is the first term? What is the sum of first two terms? What is the second term? Similarly, find the third, the tenth and the nth terms.


Answer:

Given, Sn = 4n – n2

Putting n = 1, 2


S1 = 4(1) – (1)2 = 3


S2 = 4(2) – (2)2 = 4


We know, an = Sn – Sn – 1


For first term, a1 = S1 – S0 = 3 – 0 = 3


For term a2 = S2 – S1 =4 – 3 = 1


Now fore term a3,


S2 = 4(2) – (2)2 = 8 – 4 = 4


S3 = 4(3) – (3)2 = 12 – 9 = 3


a3 = S3 – S2 = 3 – 4 = -1



Question 46.

The first and the last term of an A.P. are 17 and 350 respectively. If the common difference is 9, how many terms are there and what is their sum?


Answer:

Given,

First term, a1 = 17


Last term, an = 350 = l


And difference, d = 9


We know,


an = a + (n – 1)d


350 = 17 + (n – 1) (9)


350 = 8 + 9n


342 = 9n


n = 38


We know sum of n terms, Sn = [a + l]


S38 = [17 + 350]


= 19 * 367 = 6973


Hence, number of terms, n = 38


Sum of n terms, Sn = 6973



Question 47.

In an A.P., the first term is 2, the last term is 29 and the sum of the terms is 155. Find the common difference of the A.P.


Answer:

Given, First term, a = 2

Last term, an = l = 29


And, Sum of terms, Sn = [a + l]


155 = [2 + 29]


155 * 2 = n * 31


n = = = 10


We know, an = a + (n – 1) d


29 = 2 = (10 -1) d


29 – 2 = 9d


27 = 9d


d = 3


Hence, common difference, d = 3



Question 48.

In an A.P., the sum of first ten terms is – 150 and the sum of its next ten terms is-550. Find the A.P.


Answer:

Given: Sum of first ten terms that is, S10 = -150 and Sum of next ten terms is -550.

To find: The A.P.

Solution: Let, first term = a

Last term = an

We know in an A.P,

an = a + (n – 1) d

For n=10,

an = a + (10 -1) d

an = a + 9d

We know, Sum of n terms

Sn = [a + l] ...... (1)

l and an means the last term.

Put the value of l in (1),

So,

S10 = [a + a + 9d]

Substitute the known values,

-150 = 5 (2a + 9d)

-150 = 5 (2a + 9d)

-150= 10a + 45d

10a = -150 – 45d

a = ..... (2)


For next 10 terms, First term = a11

Last term = a20

Since an = a + (n – 1) d,

For 11th term,

a11 = a + 10d

For 20th term,

a20 = a +19d

And S'n = [a + l] ...... (3)

Let the sum of next 10 terms to be S'10 ,


Now a will be equal to a11 in this case and l will be equal to a + 19d


Put the value of a and l in the equation (3).


S’10 = [a + 10d + a + 19d]


Substitute the known values,

-550 = 5 [2a + 29d]

-550 = 10a + 145d

10a = -550 – 145d


a = .....(4)


From (2) and (4), we get


=


-150 – 45d = -550 – 145d


-45d + 145d = -550 + 150


100d = -400


d = -4


Put the value of d in (2), we get


a =


a=


We know A.P is of the form,

a1 , a2 ,.......... ,an

Where a1 is the first term of an A.P,

d is the common difference.

and

an = a+ ( n-1 )d

So,

a1 = 3

a2 = a + d = 3 + (-4) = -1

a3 = a + 2d = 3 + 2(-4) = -5

Conclusion : The A.P. is 3, -2, -5,…

and a = 3, d = -4


Question 49.

Sum of the first 14 terms of an A.P. is 1505 and its first term is 10. Find its 25th term.


Answer:

Given, a = 10

S14 = 1505


S14 = [2a + (n – 1) d]


1505 = [2(10) + (14 – 1) d]


1505 = 7 [20 + 13d]


215 = 20 + 13d


13d = 215 – 20


13d = 195


d = 15


Now, a25 = a + (n – 1) d


= 10 + (25 – 1) 15


= 10 + 24 * 15


= 10 + 360 = 370


Hence, value of 25th term is 370



Question 50.

The sum of first n terms of an A.P. is 5n2 + 3n . If its mth term is 168, find the value of m. Also, find the 20th term of this A.P.


Answer:

Given that

Sn = 5n2 + 3n


Put n = 1


S1= T1= 5 + 3 = 8


Put n = 2


S2= 5(2)2 + 3 + 2 = 26


T2= S2 – S1 = 26 – 8 = 18


S3= 5(3)2 + 3 + 3 = 54


T3= S3 – S2 = 54 – 26


= 28


Therefore, first term, a = 8 and common difference = 18 – 8 = 10


Tm = a + (m – 1) d


168 = 8 + (m – 1) 10


168 = 8 + 10m – 10


170 = 10m


m = 17


T20 = 8 + (20 – 1) 10


= 8 + 19 * 10 = 198



Question 51.

The sum of first q terms of an A.P. is 63q – 3q2. If its pth term is-60, find the value of p. Also, find the 11th term of this A.P.


Answer:

Given that: Sq= 63q – 3q2

Put q = 1


S1 = T1 = 63 – 3 = 60


Put q = 2


S2 = 63 * 2 – 3 * (2)2


= 126 – 12 = 114


T2 = S2 – S1 = 114 – 60 = 54


Put q = 3


S3 = 63 * 3 – 3(3)2


= 189 – 27 = 162


T3 = S3 – S2


= 162 – 114 = 48


Therefore, first term of this A.P. is 60 and Common difference is 54 - 60 = -6


Tp = a + (p – 1) d


-60 = 60 + (p – 1) (-6)


-120 = (p – 1) (-6)


(p – 1) = 20


p = 21


Now 11th term of this A.P


T11 = 60 + (11 – 1) (-6)


= 60 – 60 = 0



Question 52.

The sum of first m terms of an A.P. is 4 m2 - m. If its nth term is 107, find the value of n. Also, find the 21st term of this A.P.


Answer:

Sm = 4m2 – m

Put m = 1


S1 = T1 = 4 – 1 = 1


Put m = 2


S2 = 4(2)2 – 2 = 14


T2 = S2 – S1 = 14 – 3 = 11


Put m = 3


S3 = 4(3)2 – 3 = 33


T3 = S3 – S2


= 33 – 14 = 19


The first term of given A.P. is 3 and common difference, d = 11 – 3 = 8


nth term of the given A.P. is 107


107 = 3 + (n – 1)8


104 = (n – 1)8


(n – 1) = 13


n = 14


the 21st term of the given A.P., T21 = 3 + (21 – 1)8


= 3 + 160 = 163



Question 53.

The nth term of an A.P. is given by (-4n+15). Find the sum of first 20 terms of this A.P.


Answer:

Given: The nth term of an A.P. (-4n+15).

To find: the sum of first 20 terms of this A.P.

Solution:


We have,

Tn = (-4n + 15)


T1 = -4 + 15 = 11


T2 =( -4 × 2 )+ 15 = 7


T3 =( -4 × 3 ) + 15 = 3

Hence the A.P is 11,7,3,.........


The first term is 11 and the common difference is, d = T2 – T1 = 7 – 11 = -4


We calculate the sum of terms of A.P by using the formula:


Sn= [2a + (n – 1) d]


Substitute the known values to get the sum.


The sum of first 20 terms,


S20 = [(2 ×11) + (20 – 1) (-4)]



S20 = [(22) + (19) (-4)]


S20 = 10 (22 – 76)

S20 = 10(-54)

S20 = -540


Question 54.

Find the number of terms of the A.P. If 1 is added to each term of this A.P., then find the sum of all terms of the A.P. thus obtained.


Answer:

The given A.P. is -12, -9, -6,…21

Here, a = -12, d = -9 – (-12) = 3


Let the number of terms be n


Tn = a + (n – 1) d


21 = -12 + (n – 1) (3)


21 = -15 + 3n


36 = 3n


n = 12


If 1 is added to each term than A.P. is


-11, -8, -5…20


Here, a = -11, d = 3


S12= [2(-11) + (12 -1) * 3]


= 6 (-22 + 33)


= 66



Question 55.

The sum of first n terms of an A.P. is 3n2 + 4n. Find the 25th term of this A.P.


Answer:

We have sum of n terms, Sn = 3n2 + 4n

Put n = 1


S1 = T1 = 3(1)2 + 4 (1) = 7


Put n = 2


S2 = 3(2)2 + 4 (2) = 20


T2 = S2 – S1 = 20 – 7 = 13


Put n = 3


S3 = 3(3)2 + 4(3) = 39


T3 = S3 – S2 = 39 – 20 = 19


Therefore, first term is 7 and common difference, d = 13 – 7 = 6


The 25th term is, Tn= a + (n – 1) d


T25 = 7 + (25 – 1) * 6


= 7 + 24 * 6 = 151



Question 56.

In a school, students decide to plant trees in and around the school to reduce air pollution. It was decided that the number of trees, that each section of each class will plant, will be double of the class in which they are studying. If there are 1 to 12 classes in the school and each class has two sections, find how many trees were planted by the students.


Answer:

Each class has to plant double the class they are studying

Therefore, first class will plant = 2


Second class will plant = 4


Same as, 12th class will plant = 24


Total number of plants to be planted = sum of all plants


= [a + l]


= [2 + 24]


= 6 * 26 = 156


Therefore, there are 2 sections in each class


So, total trees = 2 * 156 = 312



Question 57.

The sum of first seven terms of an A.P. is 182. If its 4th and the 17th terms are in the ratio 1:5, find the A.P.


Answer:

S7 = [2a + (7 – 1) d]

182 = [2a + 6d]


52 = 2a + 6d


a + 3d = 26 (i)


Acc. To question,


a17 = 5a4


a + 16d = 5(a + 3d)


d = 4a


d = 4(26 – 3d) [from (i)]


13d = 104


d = 8


Putting the value of d in (i), we get


a = 26 – 3*8) = 2


a1 = a = 2


a2 =a + d = 2 + 8 = 10


a3 = a + 2d = 2 + 2(8) = 18


Hence, The A.P. is 2, 10, 18,…



Question 58.

The sum of the first n terms of an A.P. is 3n2 + 6n. Find the nth term of this A.P.


Answer:

Let ‘a’ be the first term and ‘d’ be the common difference

Sn= 3n2 + 6n


Firsdt term, a, S1 = 3(1)2 + 6(1)


= 3 + 6 = 9


S2 = 3(2)2 + 6(2)


a + a + d = 12 + 12


9 + 9 + d = 24


18 + d = 24


d = 6


Therefore, an = a + (n – 1) d


= 9 + (n – 1) 6


= 6n + 3



Question 59.

The sum of the first 7 terms of an A.P. is 63 and the sum of its next 7 terms is 161. Find the 28th term of this A.P.


Answer:

S7 = 63

[2(a) + 6d] = 639


(i)



Hence, for next 7 terms first term will be the 8th term i.e. a + 7d


Sum of next 7 terms, S’7 = [2(a + 7d) + 6d]


161 = 7 [a + 7d + 3d]


23 = a + 10d


23 = 9 – 3d + 10d [From (i)]


14 = 7d


d = 2


Putting the value of d in (i), we get


A = 9 – 3(2) = 3


Now, a28= a + 27d


= 3 + 27(2)


= 3 + 54 = 57



Question 60.

If demotes the sum of the first n terms of an A.P., prove that S30= 3 (S20 – S10) .


Answer:

Proof: S20 = [2a + 19d]

= 10 [2a + 19d] (i)


S10 = [2a + 9d]


= 5 [2a + 9d] (ii)


L.H.S: S30 = [2a + 29d]


= 15 [2a + 29d]


R.H.S: 3 (S20 – S10)


= 3 [10 (2a + 19d) – 5 (2a + 9d)] {From (i) and (ii)}


= 15 [4a + 38d – 2a – 9d]


= 15 [2a + 29d]


Since, L.H.S = R.H.S


Hence, proved



Question 61.

The sum of first n terms of an A.P. is 5n – n2. Find the nth term of this A.P.


Answer:

Sn = 5n – n2

First term, a = 5(1) – (1)2


= 5 – 1 = 4


S2 = 5(2) – (2)2


a + (a + d) = 10 – 4


4 + 4 + d = 6


d = -2


Now, the nth term, an = a + (n - 1) d


= 4 + (n – 1) (-2)


= 4 + 2 – 2n = 6 – 2n



Question 62.

The sum of the first n terms of an A.P. is 4n2 + 2n. Find the nth term of this A.P.


Answer:

Sn = 4n2 + 2n

First term, a = 4(1)2 + 2(1)


= 4 + 2 = 6


Sum of first two terms, S2 = 4(2)2 + 2(2)


a + a + d = 16 + 4


6 + 6 + d = 20


d = 8


Now the nth term, an = a + (n – 1) d


= 6 + (n – 1) 8


= 6 + 8n – 8


= 8n - 2



Question 63.

If the 10th term of an A.P. is 21 and the sum of its first ten terms is 120, find its nth term.


Answer:

a10 = a + 9d

a + 9d = 21 (i)


S10 = [2a + 9d]


120 = 5 [2a + 9d]


24 = 2a + 9d


24 = 2 (21 – 9d) + 9d [From (i)]


-18 = -9d


d = 2


Putting the value of d in (i), we get


a = 21 – 9(2)


= 3


The nth term, an = a + (n – 1) d


= 3 + (n – 1) 2


= 3n + 2n – 2


= 2n + 1



Question 64.

Ram kali would need Rs.1800 for admission fee and books etc., for her daughter to start going to school from next year. She saved Rs.50 in the first month of this year and increased her monthly saving by Rs.20. After a year, how much money will she save? Will she be able to fulfill her dream of sending her daughter to school?

[CBSE2005,2014]


Answer:

a = 50, d = 20

an = a + (n – 1) d = l


a12 = 50 + 11 * 20 = 270


Sn = [a + l]


= [50 + 270]


= 12 * 160 = 1920


Yes, she will be able to fulfill her dream of sending her daughter to school.



Question 65.

The first and the last terms of an A.P. are 5 and 45 respectively. If the sum of all its terms is 400, find its common difference.


Answer:

First term, a = 5

Last term, l = 45


Sn = 400


[a + l] = 400



n = 16


Now, n = + 1


16 = + 1


16d = 40 + d


15d = 40


d =


Hence, the common difference is



Question 66.

The first and the last terms of an AP are 7 and 49 respectively. If sum of all its terms is 420, find its common difference.


Answer:

First term, a = 7

Last term, l = 49


Sn = [a + l]


420 = [7 + 49]


840 = 56n


n = 15


Now, n = + 1


15 = + 1


15d = 42 + d


14d = 42


d = 3


Hence, it’s common difference is 3



Question 67.

If demotes the sum of first n terms of an A.P., prove that S12 = 3 (s8 – S4) .


Answer:

S8 = [2a + 7d]

= 4 (2a + 7d) (i)


S4 = [2a + 7d]


= 2 (2a + 7d) (ii)


L.H.S = S12 = [2a + 11d]


= 6 [2a + 11d]


R.H.S = 3 (S8 – S4)


= 3 [4 (2a + 7d) – 2 (2a + 3d)] [From (i) and (ii)]


= 6 [4a + 14d – 2a – 3d]


= 6 [2a + 11d]


Since, L.H.S = R.H.S


Hence, proved



Question 68.

If the sum of first n terms of an A.P. is (3n2 + 7n), then find its nth term. Hence write its 20th term.


Answer:

Sn = (3n2 + 7n)

First term, a = S1 = [3(1)2 + 7(1)]


= [3 + 7] = 5


S2 = [3(2)2 + 7(2)]


a + a + d = [12 + 14]


5 + 5 + d = 13


d = 3


an = a + (n – 1)d


= 5 + (n – 1) 3


= 5 + 3n – 3


= 3n + 2


a20 = 3(20) + 2


= 60 + 2 = 62


Hence, it’s 20th term is 62



Question 69.

The sum of first 9 terms of an A.P. is 162. The ratio of its 6th term to its 13th term is 1:2. Find the first and 15th term of the A.P.


Answer:

Let a be the first term and d be the common difference

Now, a6 / a13 =


=


2a + 10d = a + 12d


a = 2d (i)


Now sum of first n terms of A.P


Sn = [2a + (n – 1) d]


S9 = [2a + 8d]


162 = 9 (a + 4d)


a + 4d = 18


2d + 4d = 18 [Using (i)]


6d = 18


d =3


Now from (i), we get


a = 2 * 3 = 6


So, first term, a1 = a = 6


15th term, a15 = a + 14d = 6 + 14(3)


= 6 + 42 = 48


Therefore, First term is 6 and 15th term is 48




Cce - Formative Assessment
Question 1.

Define an arithmetic progression.


Answer:

An arithmetic progression is a list of numbers in which each term is obtained by adding a fixed number to the preceding term except the first term. This fixed number is called the common difference of the AP.



Question 2.

If 7th and 13th terms of an A.P. be 34 and 64 respectively, then its 18th term is
A. 87

B. 88

C. 89

D. 90


Answer:

Here, a7 = 34


a13 = 64


a7 = a + 6d = 34……….(1)


a13 = a + 12d = 64…………(2)


Subtracting (1) from (2)


6d = 30


D = 5


Multiplying (1) by 2


2a + 12d = 68………………….(3)


Subtracting (2) from (3)


a = 4


a18 = a + (n–1) d


a18 = 4 + (17) 5


a18 = 89 = C


Question 3.

Write the common difference of an A.P. whose nth term is an = 3n + 7.


Answer:

If an = 3n + 7

Then a1 = 3 (1) + 7 = 10


a2 = 3(2) + 7 = 13


a3 = 3(3) + 7 = 16


d= common difference = an – a n–1


= a3 – a2


= 16 – 13 = 3



Question 4.

If the sum of P terms of an A.P. is q and the sum of q terms is p, then the sum of p + q terms will be
A. 0

B. p – q

C. p + q

D. – (p + q)


Answer:

Let a be the first term and d is common difference of the A.P


then sum of n terms in A.P is Sn = ()[ 2a + (n – 1) d]


Here, sp= q


Sq =p


Sp = () [2a + (p – 1) d]


q = [2a + (p – 1) d]


= [2a + (p – 1) d] --------(1)


Sq = p = ( [2a + (q – 1) d]


= [2a + (q – 1) d] …………..(2)


Subtract (1) from (2) we get



(q – p) d = / pq -----------(3)


d = -2(q + p) / pq -----------(3)


Sum of first (p + q) terms




[from (1) and (3)]





Question 5.

Which tem of the sequence 114, 109, 104, .... is the first negative term?


Answer:

Here a = 114, d is common difference

d = a3 – a2 = a2 – a1 = –5


For finding first negative term


Tn < 0


a + (n – 1) d < 0


114 + (n – 1) (– 5) < 0


114 – 5n + 5 < 0


119 – 5n < 0


– 5n < – 119


5n > 119


n > 119/5


n > 23.8


Therefore first negative term is 24th term



Question 6.

If the sum of n terms of an A.P. be 3n2 + n and its common difference is 6, then its first term is
A. 2

B. 3

C. 1

D. 4


Answer:

Here, Sn= 3n2 + n


d = 6


Putting n= 1


S1 = 3 + 1 = 4


Sum of first 1 term = first term = 4


Question 7.

The first and last terms of an A.P. are 1 and 11. If the sum of its terms is 36, then the number of terms will be
A. 5

B. 6

C. 7
D. 8


Answer:

a1 = 1 (first term)


a11 = 11 = l (last term)


Sn= 36


But, Sn = (a + l)


36= (12)


n= 6


Question 8.

Write the value of a30 – a10 for the A.P. 4, 9, 14, 19, ……….


Answer:

Here a1 = 4

a2 = 9


a3 = 14


d = common difference = a3 – a2 = a2 – a1 = 14 – 9 = 9 – 4 = 5


a30 = a + (n–1) d


a30 = 4 + (30 –1) 5


a30= 4 + 29 X 5


a30= 149


Similarly, a10 = 4 + 9x5


a10= 49


a30 – a10 = 149 – 49 = 100



Question 9.

If the sum of n terms of an A.P. is 3n2 + 5n then which of its terms is 164?
A. 26th

B. 27th

C. 28th

D. none of these


Answer:

Here, Sn = 3n2 + 5n


S1 = a1 = 3 + 5 = 8


S2 = a1 + a2 = 12 + 10 = 22


⇒ a2 = S2 – S1 = 22 – 8 = 14


S3 = a1 + a2 + a3 = 27 + 15 = 42


⇒a3 = S3 – S2 = 42 – 22 = 20


∴ Given AP is 8, 14, 20, .....


Thus a = 8, d = 6


Given tm = 164.


164 = [a + (n –1)d]


164 = [(8) + (m –1)6]


164 = [8 + 6m – 6]


164 = [2 + 6m]


162 = 6m


m = 162 / 6.


∴ m = 27.


Question 10.

Write 5th term from the end of the A.P. 3, 5, 7, 9, ...., 201.


Answer:

Here a = 201, a2 = 5, a3 = 7

d = a3 – a2 = a2 – a1


= 7 – 5 = 5 – 3 = 2


tn = a + (n –1)d


tn = 3 + (n–1)2 = 201


(n –1) 2 = 201 – 3 = 198


n – 1 = = 99


n= 99 + 1 =100


5th term from end = 96th term


T95 = 3 + (96 –1)2


T95 = 3 + 95 x 2


T95 = 3 + 190 = 193



Question 11.

If the sum of n terms of an A.P. is 2n2 + 5n, then its nth term is
A. 4n – 3

B. 3n – 4

C. 4n + 3

D. 3n + 4


Answer:

Here Sn = 2n2 + 5n


Sum of the A.P with 1 term = S1 = 2 + 5 = 7 = first term


Sum of the A.P with 2 terms = 8 + 10 = 18


Sum of the A.P with 3 terms = 18 + 15 = 33


a2 = S2 – S1 = 18 – 7 = 11


d = a2 –a1 = 11 – 7 = 4


nth term = a + (n–1) d


= 7 + (n–1) 4


nth term = 4n + 3


Question 12.

Write the value of x for which 2x, x + 10 and 3x + 2 are in A.P.


Answer:

Terms are in A.P. if common difference (d) is same between two continuous numbers.

x + 10 – 2x = 3x + 2 – (x + 10)


10 – x = 2x – 8


18 = 3x


x= 6


So the terms in A.P. are: 2(6), 6 + 10, 3(6) + 2


= 12, 16, 20



Question 13.

If the sum of three consecutive terms of an increasing A.P. is 51 and the product of the first and third of these terms is 273, then the third term is
A. 13

B. 9

C. 21

D. 17


Answer:

Let 3 consecutive terms A.P is a –d, a, a + d. and the sum is 51


So, (a –d) + a + (a + d) = 51


3a –d + d = 51


3a = 51


a = 17


The product of first and third terms = 273


So, (a –d) (a + d) = 273


a2 –d2 = 273


172 –d2 = 273


289 –d 2 = 273


d2 = 289 –273


d2 = 16


d = 4


Third term = a + d = 17 + 4 = 21


Question 14.

Write the nth term of an A.P. the sum of whose n terms is Sn.


Answer:

First term = a

Sum up to first term = a


Last term (nth term) = an


Sum up to n terms = Sn


Second last term = an–1


Sum up to (n–1) th term = sn–1


Therefore, an = Sn–Sn–1



Question 15.

If four numbers in A.P. are such that their sum is 50 and the greatest number is 4 times the least, then the numbers are
A. 5, 10, 15, 20

B. 4, 10, 16, 22

C. 3, 7, 11, 15

D. none of these


Answer:

Let the 4 numbers be a, a + d, a + 2d, a + 3d.


Sum of 4 numbers in A.P = 50


a + a + d + a + 2d + a + 3d = 50


⇒ 4a + 6d = 50


⇒ 2a + 3d = 25 --------------(1)


Given the greatest number is 4 times the least.


4(a) = a + 3d


4a – a = 3d


a = d


Putting, a = d in (1), we obtain


5d = 25


d = 5


⇒a = 5


∴ First four terms are 5, 10, 15, 20.


Question 16.

Write the sum of first n odd natural numbers.


Answer:

First n odd numbers are 1, 3, 5, 7, 9........., (2n–1) which forms an A.P

a= 1


l= 2n – 1 where l be the last term


Sn = (2a + (n–1) d) or (a + l)


= (1 + 2n –1)


= (2n)


=n (n)


=n2



Question 17.

Let Sn denote the sum of n terms of an A.P. whose first term is a. If the common difference d is given by d = Sn – kSn–1 + Sn–2, then k =
A. 1

B. 2

C. 3

D. none of these.


Answer:

Let a be the first term, n be the number of terms and d be the common difference of AP.


Given d = Sn – kSn–1 + Sn–2.


Now let n = 3


So, AP is : a, a + d, a + 2d


And d = S3 – k S3–1 + S3–2


d = S3 – k S2 + S1 ..............(1)


Sum of n terms of an AP is given as:


Sn = () x {2a + (n–1) d}


Now S1 = a


S2 = () x (2a + (2–1) d) (n =2)


S2 = (2a + d)


S3 = () x (2a + (3–1)d) (n =3)


S3 = x (2a + 2d)


S3 = 3(a + d)


S3 = 3a + 3d


Putting values of S1, S2 and S3 in equation 1, we get


d = 3a + 3d – k (2a + d) + a


d = 4a + 3d – k (2a + d)


k (2a + d) = 4a + 3d – d


k (2a + d) = 4a + 2d


k (2a + d) = 2(2a + d)


k = 2


Question 18.

Write the sum of first n even natural numbers.


Answer:

First n even Natural numbers are 2, 4, 6,……, 2n

This forms an A.P where


a= 2


l= 2n where l is last term of the A.P


Sn = 𝑛/2 (2a + (n–1) d) or (a + l)


= (2 + 2n)


= [2(1 + n)]


=n (n + 1)



Question 19.

The first and last term of an A.P. are a and l respectively. If S is the sum of all the terms of the A.P. and the common difference is given by , then k=
A. S

B. 2S

C. 3S

D. none of these


Answer:

Let the common difference and number of terms of AP be d and n respectively.


Last term of AP = an = l (given)


l = a + (n–1) d


n = + 1……………… (1)


S= (2a + (n–1) d)


S = + 1) (2a + ( + 1 –1) d) …………… using (1)


S= + 1) (2a + l –a)


S= + 1) (a + l)


( + 1) =


= –1


=


D =


Comparing this with


We get k =2S


Question 20.

1f the sum of n terms of an A.P. is Sn = 3n2 + 5n. Write its common difference.


Answer:

Let a1, a2, a3…….. an be the given A.P

Given, sum of n terms = 3n2 + 5n


Sn=3n2 + 5n……….(1)


Putting n =1 in (1)


Sn= 3x12 + 5x1


= 3 + 5 = 8


Sum of first 1 terms = first term


first term = a = S1 = 8


Sn = 3n2 + 5n


Putting n= 2 in….. (1)


S2= 3x 22 + 5x2


S2= 22


Sum of first two terms = first term + second term


S2= a1 + a2


S2 – a1 = a2


a2= 22 – 8 = 14


Thus a1= 8, a2 = 14


d= common difference = 14 – 8= 6



Question 21.

Write the expression for the common difference of an A.P. whose first term is a and nth term is b.


Answer:

The nth term of the A.P whose first term a1 and common difference is d is given by

an = a + (n–1) d


Here an is given as b


so b = a + (n–1) d


= d


⇒ d=



Question 22.

If the sum of first n even natural numbers is equal to k times the sum of first n odd natural numbers, then k =
A.

B.

C.

D.


Answer:

Given: the sum of first n even natural numbers is equal to k times the sum of first n odd natural numbers.

To find: The value of k

Solution:

Sum of terms of A.P = n/2 ( 2a + (n-1) d )

First n even natural numbers are: 2, 4, 6, 8, ............


It forms an AP where first term a = 2 and common difference d = 4 – 2 = 2


⇒ sum of n even natural terms = x {2 x 2 + (n–1)2}

= ( 4 + 2n - 2 )

= ( 2 + 2n )

= × 2 ( 1 + n )

= n (n + 1)


First n odd natural numbers are: 1, 3, 5, 7, ............


It forms an AP where first term is a = 1 and the common difference d = 3 – 1 = 2


Now sum of n terms = {2 x 1 + (n–1)2}


= ( 2 + 2n - 2 )

= (2n )

= n x n

= n2


Now, According to given condition


Sum of first n even numbers = k X (Sum of first n odd numbers)


⇒ n (n + 1) = k x n2


⇒ k x n = n + 1


⇒ k =


Question 23.

If the first, second and last term of an A.P. are a, b and 2a respectively, its sum is
A.

B.

C.

D. none of these


Answer:

a, b and 2a are in A.P so a is the first term and 2a is the last term denoted by T and Tn respectively. Here Common difference = b – a


Tn = 2a = a + (n–1) (b–a)


So n =


Sum = {first term + last term}


={ 3a }


=


Question 24.

The first term of an A.P. is p and its common difference is q. Find its 10th term.


Answer:

Given: The first term of an A.P. is p and its common difference is q.

To find: its 10th term.

Solution:

The first term of A.P is p.

Common difference = q

We know an = a + (n–1)d

Where a is first term and d is common difference.

So,

a10 = p + (10-1)q

a10 = p + 9q


Question 25.

For what values of p are 2p + 1, 13, 5p – 3 are three consecutive terms of an A.P.?


Answer:

Given: 2p + 1, 13, 5p – 3 are three consecutive terms of an A.P.

To find: The value of p

Solution:

Consider a1 = 2p + 1 , a2 = 13 and a3 = 5p – 3

The A.P will be of the form

2p + 1, 13, 5p – 3,.........

Since the terms are in A.P so the common differences in them is same.

⇒ a2 - a1 = a3 - a2

⇒ 13– (2p + 1) = 5p –3 –13

⇒ 13– 2p - 1 = 5p –3 –13

⇒12– 2p = 5p –16

⇒12+16 = 5p +2p

⇒ 28 = 7p

⇒ p= 4


Question 26.

If S1 is the sum of an arithmetic progression of 'n' odd number of terms and S2 the sum of the terms of the series in odd places, then =
A.

B.

C.

D.


Answer:

S1 = (2a + (n–1) d)


Out of these odd numbers of terms, there are terms in odd places


S2 = (2a + ( –1) d)


Common difference of two odd places is 2d


S2 = (2a + (n–1) d)


Now,


=



Question 27.

If , a, 2 are three consecutive terms of an A.P., then find the value of a.


Answer:

, a, 2 are three terms in an A.P so there common difference will be same

a– = 2–a


2a= 2 +


2a=


a =



Question 28.

If in an A.P., Sn = n2p and Sm = m2p, where Sr denotes the sum of r terms of the A.P., then Sp is equal to
A.
B. mn p

C. p3

D. (m + n) p2


Answer:

Let first term = a and Common difference = d


∴ According to the question, S n = n 2 p


Sn = n/2 (2a + (n–1) d) = n2p


2a + (n–1) d = 2np……………….(1)


And Sm = m2p


Sm= m/2 (2a + (m–1) d) = m2p


2mp = (2a + (m–1) d)……………….(2)


Subtracting 2 from 1


2a + (n–1) d – 2a – (m–1) d = 2 np – 2 mp


d (n–1 –m + 1) = 2p (n– m)


d = 2p


putting value of d in (1)


2a + (n–1) 2p = 2np


a + (n–1)p = np


a = p


now Sp = p/2 (2a + (p–1) d)


putting value of a = p and d = 2p


Sp = p/2 (2p + (p–1) 2p)


Sp = p3


Question 29.

If Sn denote the sum of the first n terms of an A.P. If S2n = 3Sn, then S3n : Sn is equal to
A. 4

B. 6

C. 8

D. 10


Answer:

We know that Sn =


Now it is given that


S2n = 3Sn


= 3)


2(2n + 1) = 3(n + 1)


4n + 2 = 3n + 3


n = 1


now, S3n/Sn = ()


=


= 6


Question 30.

If the sum of first p term of an A.P. is ap2 + bp, find its common difference.


Answer:

Here, sp = ap2 + bp

S1= a + b


S2= ax22 + bx2


=4a + 2b


S2 = a1 + a2


4a + 2b = a1 + a2


A2 = 4a + 2b – (a + b)


A2 = 3a + b


Now, d = common difference = a2 – a1


= 3a + b – (a + b)


= 2a



Question 31.

In an AP, Sp = q, Sq = p and Sr denotes the sum of first r terms. Then, Sp + q is equal to
A. 0

B. –(p + q)

C. p + q

D. pq


Answer:

Sp = {2A + (p–1) D} = q, given


Sq = (2A + (q–1) D} = p, given


On Subtracting the second equation from 1st we get,


D = –2


Also On adding the two equations we get,


2A + = p/q + q/p


Now,


Sp + q = {2A + (p + q–1) D}


= {2A + (p + q–1)D}


{ + + (p + q–1) D}


=


[ By substituting the value of D ]


= – (p + q)


Question 32.

If Sr denotes the sum of the first r terms of an A.P. Then, S3n: (S2n — Sn) is
A. n

B. 3n

C. 3

D. none of these


Answer:

S2n = (2a + (2n–1) d)


Sn = (2a + (n–1)d)


S3n = (2a + (n–1)d)


S2n –Sn = (2a + (2n–1) d) – (2a + (n–1)d)


= (2a + (n –1) d)


S3n: (S2n — Sn) = (2a + (n–1)d) : (2a + (n –1) d)


= 3


Question 33.

If the first term of an A.P. is 2 and common difference is 4, then the sum of its 40 terms is
A. 3200

B. 1600

C. 200

D. 2800


Answer:

Here a1= 2


D = common difference = 4


Sn = (2a + (n–1) d)


S40 = (2 x 2 + (39) 4)


= 20 (4 + 156)


= 3,200


Question 34.

The number of terms of the A.P. 3, 7, 11, 15, ... to be taken so that the sum is 406 is
A. 5

B. 10

C. 12

D. 14


Answer:

Here a = 3


a2= 7


a3 = 11


d= a3– a2 = a2– a1 = 4


Sn= (2a + (n–1) d)


406 = (6 + (n–1)4)


406 = (4n + 2)


2n2 + n – 406 = 0


2n2 + 29 n – 28 n – 406 = 0


n (2n + 29) – 14(2n + 29) = 0


(n – 14) (2n + 29) = 0


Number of terms cannot be negative and in fractions so n= 14


Question 35.

Sum of n terms of the series is
A.

B. 2n (n + 1

C.

D. 1


Answer:

The given A.P is


The simplified A.P is



Here a=


d= =


Sn = (2a + (n–1) d)


= (2 + (n–1))


= /2 (1 + n)


= n (n + 1)/


Question 36.

The 9th term of an A.P. is 449 and 449th term is 9. The term which is equal to zero is
A. 501th

B. 502th

C. 508th

D. none of these


Answer:

Here, a9 = 449


A449 = 9


Let a is the first term and d is the common difference of the AP


Given 9th term of AP = 499


a + 8d = 499 .....(1)


Again 499th term of AP = 9


a + 498d = 9 .....(2)


Now subtract equation 1 and 2, we get


a + 8d – (a + 498d) = 499 – 9


a + 8d – a – 498d = 499 – 9


–490d = 490


d = –490/490


d = –1


Put value of d in equation1, we get


a – 8 = 499


a = 499 + 8


a = 507


Let nth term is equal is to zero


a + (n–1)d = 0


507 – (n–1) = 0 (By putting value of a and d)


507 – n + 1 = 0


508 – n = 0


n = 508


So 508th term of AP is zero.


Question 37.

If are in A.P. Then, x =
A. 5

B. 3

C. 1

D. 2


Answer:

Here are in A.P


so



–1/ = –2/


1/2 = (x + 2)/(x + 5)


x + 5= 2x + 4


x= 1


Question 38.

The nth term of an A.P., the sum of whose n terms is Sn, is
A. Sn + Sn–1

B. Sn – Sn–1

C. Sn + Sn + 1

D. Sn – Sn + 1


Answer:

The sum of n terms of an A.P is given by Sn


Sn = (2a + (n–1) d)


If the sum of n terms is given that is Sn is given then the nth term is given by the formula


Tn = Sn– Sn–1


Where Sn–1 is sum of the (n–1)th term of the A.P.


Question 39.

The common difference of an A.P., the sum of whose n terms is Sn, is
A. Sn – 2Sn–1 + Sn–2

B. Sn – 2Sn–1– Sn–2

C. Sn – Sn–2

D. Sn – Sn–1


Answer:

an is the nth term of an A.P and a n–1 is the (n–1)th term of an A.P,


d = common difference, Sn = sum of n terms of an A.P


d= an – an–1


But an= Sn – Sn–1


And an–1= Sn–1 – Sn–2


So d= Sn – Sn–1 – (Sn–1 – Sn–2)


d= Sn – 2 Sn–1 + Sn–2


Question 40.

If the sums of n terms of two arithmetic progressions are in the ratio , then their nth terms are in the ratio
A.

B.

C.

D.


Answer:

The sums of n terms of two A.P`s are in ratio


Let a1 and d1 be the first term and common difference of the first A.P, respectively. Similarly let a2 and d2 be the common difference of the second A.P, respectively.


According to the given condition


=


=


=


Now the Nth term is given by a + (N–1) d


Equating the coefficients



⇒ n = 2N – 1


= = =


Question 41.

If Sn, denote the sum of n terms of an A.P. with first term a and common difference d such that , is independent of x, then
A. d=a

B. d=2a

C. a = 2d

D. d = – a


Answer:

Given AP in which First term = a, Common difference = d, Number of terms = n


And Sn denotes the sum of n terms


So



and


Now,






If d= 2a


Sx /Skx = (2×a) / (k2×2a)


= 1/ k2


So Sx /Skx is independent ofif d = 2a


Question 42.

If the first term of an A.P. is a and nth term is b, then its common difference is
A.

B.

C.

D.


Answer:

‘a’ is the first term of the A.P,


‘b’ is the (n)th term and ‘d’ is the common difference of the A.P.


Then, we have b = a + (n–1) d


= a + (n + 1)d


Hence, ‘d’ =


Question 43.

The sum of first n odd natural numbers is
A. 2n – 1

B. 2n + 1

C. n2

D. n2 – 1


Answer:

The sum of the first n odd numbers forms an arithmetic progression with first term equal to 1 and the last term equal to 2(n−1) + 1


The formula for an arithmetic progression of n terms with first term a1 and last term an is


(a1 + an)


Substituting these values, a1=1, an= 2(n−1) + 1


⇒ n (1 + 2n−2 + 1)2 = n (2n) 2= n2


Question 44.

Two A.P.'s have the same common difference. The first term of one of these is 8 and that of the other is 3. The difference between their 30th terms is
A. 11

B. 3

C. 8

D. 5


Answer:

for 1st A.P a = 8 and d= common difference = d


The nth term of an A.P is given by the formula


an= a + (n–1) d


The required difference is


= 8 + (30 – 1) d – 3 – (30 –1) d


= 5


Question 45.

If 18, a, b, –3 are in A.P., the a + b =
A. 19

B. 7

C. 11

D. 15


Answer:

Given 18, a, b, –3 are in A.P


a– 18 = –3 – b


Because the common difference of the A.P. always remains the same


a + b = 15


Question 46.

The sum of n terms of two A.P.'s are in the ratio 5n + 9 : 9n + 6. Then, the ratio of their 18th term is
A.

B.

C.

D.


Answer:

Let a1 and a2 be the first terms of the two A.P. and d1 and d2 be the common difference of the two A.P


According to the given condition,



Substituting n= 35 in……… (1)



an = a + (n–1) d



Or 179: 321


Question 47.

If , then n =
A. 8

B. 7

C. 10

D. 11


Answer:


Now, Sum of n terms



=


= n[5 + 2n–2]


=n[3 + 2n]


Now,



=


= (n + 1)[7 + n]




On cross multiplying we get,


16n[3 + 2n] = 17n + 17[7 + n]


⇒ 48n + 32n2 = 119n + 17n2 + 119 + 17n


⇒ 48n + 32n2 = 136n + 17n2 + 119


⇒ 15n2 – 88n – 199 = 0


⇒ n =


Question 48.

The sum of n terms of an A.P. is 3n2 + 5n, then 164 is its
A. 24th term

B. 27th term

C. 26th term

D. 25th term


Answer:

S = 3n2 + 5n
S1 = a1 = 3 + 5 = 8
S2 = a1 + a2 = 12 + 10 = 22
⇒ a2 = S2 – S1 = 22 – 8 = 14
S3 = a1 + a2 + a3 = 27 + 15 = 42
⇒ a3 = S3 – S2 = 42 – 22 = 20
∴ Given AP is 8,14,20,.....
Thus a = 8, d = 6
Given tm = 164.
164 = [a + (n –1)d]
164 = [(8) + (m –1)6]
164 = [8 + 6m – 6]
164 = [2 + 6m]
162= 6m
m = 162 / 6.
∴ m = 27.


Question 49.

If the nth term of an A.P. is 2n + 1, then the sum of first n terms of the A.P. is
A. n (n – 2)

B. n (n + 2)

C. n (n + 1)

D. n (n –1)


Answer:

an= a + (n–1) d = 2n + 1 (given)…………………(1)


Sn= n/2 (2a + (n–1) d)


By putting n=1 in (1)


a=3


similarly a2= 5


a3= 7


d= common difference = a2 – a1= 2


Sn = n/2 (6 + (n–1) 2)


= n/2(2n + 4)


= n (n + 2)


Question 50.

If 18th and 11th term of an A.P. are in the ratio 3 : 2, then its 21st and 5th terms are in the ratio
A. 3: 2

B. 3: 1

C. 1 :3

D. 2 : 3


Answer:

As 18th term : 11th term ratio is 3:2


18th term is a + 17d and 11th term is a + 10d



3a + 30d = 2a + 34d


a=4d…………….(1)


21st term is a + 20d = 24d (putting value of a from (1))


And 5th term is a + 4d= 8d (putting value of a from (1))


Ratio of 21st term : 5th term is


= 3:1


Question 51.

The sum of first 20 odd natural numbers is
A. 100

B. 210

C. 400

D. 420


Answer:

Here A.P is 1,3, 5, ……


a= 1, d= 2 and n = 20


Sn = (2a + (n–1) d)


Sn = 20/2 (2 x 1 + (19) 2)


Sn = 10 (2 + 38)


Sn= 400


Question 52.

The common difference of the A.P. is , is
A. –1

B. 1

C. q

D. 2q


Answer:

Here A.P =


d= a2– a1


d


d=


d= –1


Question 53.

The common difference of the A.P. is
A.

B.

C. –b

D. b


Answer:

A.P =


d = common difference = a2–a1


=


d=


d= –b


Question 54.

The common difference of the A.P. is
A. 2b

B. –2b

C. 3

D. –3


Answer:

Here A.P is


d= a2–a1


d=


d=


d= –3


Question 55.

If k, 2k –1 and 2k + 1 are three consecutive terms of an AP, the value of k is
A. – 2

B. 3

C. – 3

D. 6


Answer:

Here A.P = k, 2k –1, 2k + 1


Since the numbers are in A.P their common difference (d) should be same


d=a2–a1 = a3–a2


2k–1–k = 2k + 1– (2k –1)


k– 1 = 2


k = 3


Question 56.

The next term of the A.P.
A.

B.

C.

D.


Answer:

A.P is


The simplified A.P is√7, 2√7 3√7


d= 2√7 – √7 = √7 (2–1) = √7


Next term of A.P means the fourth term = a4


a4 = a + (n–1) d


a4= √7 + 3√7


= 4√7


= √112


Question 57.

The first three terms of an A.P. respectively are 3y – 1, 3y + 5 and 5y + 1. Then, y equals
A. –3

B. 4

C. 5

D. 2


Answer:

A.P here is 3y – 1, 3y + 5 and 5y + 1.


So d= common difference = a2– a1= a3–a2


3y + 5 – (3y–1) = (5y + 1) – (3y + 5)


6 = 2y – 4


10 / 2 = y


Y = 5